Сопротивление человека электрическому току: сопротивление человека электрическому току от чего зависит

Содержание

сопротивление человека электрическому току от чего зависит

Стоит человеку случайно (или намеренно?) прикоснуться к электрическим проводам или каким-либо другим элементам соответствующего оборудования, по которому протекает электрический ток, как немедленно его тело «включается» в систему передачи электричества по проводникам. Результат – электрические травмы разной степени тяжести и, к сожалению, нередки случаи летального исхода.

Тяжесть и степень негативного воздействия на организм таких электротравм напрямую зависит от целого ряда различных параметров, таких как технические характеристики самой сети, состояние человеческого тела и т.п. Известно, что человеческое тело, как минимум, на 70% состоит из влаги. Это приводит к тому, что перенос заряженных электрических частиц осуществляется не электронами, а ионами. Электроны действуют в металлических проводниках, а тело человека, считающееся особенным видом проводника и является особым видом проводника.

Это не что иное, как электролит, обладающий переменным сопротивлением.

Общие понятия процесса

В соответствии с основным законом физики, при прохождении через тело человека, его внутренние органы, кожный покров и т.п. электрического тока, оно (тело) начинает оказывать определенное сопротивление, как и всякий другой проводник. Однако, в отличие от многих других видов проводников, прежде всего – металлов, у который величина сопротивления имеет более-менее постоянные показатели и меняется только в соответствии с изменением силы тока, диаметра проводника и некоторых других внешних факторов, тело человека работает по-другому.

Прохождение электрического тока через ткани человека – это достаточно сложный процесс, в котором тесно переплетены биохимические и биофизические составляющие показатели. Эти параметры характерны исключительно для

живой материи. Таким образом величина сопротивления тела — это параметр переменный, меняющийся с учетом конкретных внутренних и внешних условий. В том числе, замеры могут показать существенную разницу для определенных частей тела – рук, ног, туловища или головы. Таким образом можно установить, что сопротивление человека электрическому току зависит от целого ряда параметров, а каких именно – узнаем чуть позже.

Состояние кожного покрова

Кожа покрывает практически все тело и именно поэтому она первой испытывает нагрузку при столкновении человека с электричеством. По этой причине общее сопротивление организма разряду во многом напрямую зависит именно от степени сопротивляемости самой кожи. Рассмотрим кратко строение кожного покрова. У любого человека это 2 основных слоя:

  • Верхний (эпидермис), состоит из 5 слоев, общей толщиной не более 0,12 мм;
  • Внутренний (дерма), здесь толщина намного больше и может достигать 2 мм.

Самую высокую сопротивляемость у всего дела оказывает именно верхний стой кожи. Эпидермис не содержит капилляров своем составе, что и обуславливает его повышенную устойчивость к таким видам электрических нагрузок.

Все остальные слои кожи менее устойчивы к такому воздействию (и это также обусловлено структурой данных слоев).

Таким образом, если рассматривать сопротивление кожного покрова, то общий показатель здесь – это сумма двух параметров: сопротивления, которое оказывает эпидермический слой с каждой стороны тела, а также сопротивление слоя дермы + внутренности организма.

Основные параметры

Специалисты, исследуя влияние электричества на тело человека, зафиксировали целый ряд параметров, которые непосредственно оказывают влияние на реакцию организма при подобных негативных видах воздействия. Сопротивление тела человека электрическому току зависит от:

  • Качественного состояния кожного покрова;
  • Точного места, где в тело человека начал поступать ток;
  • Основных физических параметров электросети, а именно – показатели тока и напряжения;
  • Периода времени (длительность) в течение которого продолжалось воздействие тока на организм;
  • Определенных параметров состояния окружающей среды во время и в месте воздействия электрического тока на организм и его прохождение через тело человека (уровень влажности, температура и т. п.).

Важное дополнение

Если на коже у человека имеются какие-либо повреждения – раны (тем более, открытые), царапины, порезы, ожоги и т.п., сопротивляемость верхнего слоя резко снижается. Как следствие – более тяжелая степень негативного воздействия на организм в целом.

Подведем итог

Теперь мы знаем, от чего зависит тяжесть поражения электрическим током человека. Важно понимать, что пусть тело и является неплохим проводником электричества, но имеет определенный запас сопротивления электротоку. Это показатели могут существенно меняться в зависимости от конкретных условий.

Соблюдение техники безопасности – важнейшее условие и главное требование для эффективной трудовой деятельности и сохранения здоровья и самой жизни!

Чему равно сопротивление тела человека

Сопротивление тела человека

Электрическое сопротивление различных тканей тела человека не одинаково. Например, при токе частотой 50 Гц удельное сопротивление равно: кости – 107 Ом∙м, кожа сухая – 105 Ом∙м, крови – 1,7 Ом∙м. При сухой, чистой и неповрежденной коже сопротивление тела, измеренное, при напряжении 15-20 В переменного тока (50 Гц), колеблется в пределах от 1 до 10 кОм, а иногда и в более широких пределах.

Сопротивление кожи, а следовательно сопротивление тела в целом резко уменьшается при повреждении ее рогового слоя, наличие влаги на ее поверхности, интенсивном потовыделении и загрязнении.

Электрическое сопротивление тела человека зависит так же от места приложения электродов к телу, значений тока, проходящего через человека, и приложенного к телу напряжения, рода и частоты тока, площади электродов, длительности прохождения тока через человека и некоторых других факторов. Увеличение тока приводит к снижению сопротивления соответствующих участков кожи, за счет местного нагрева кожи и действия на центральную нервную систему (усиливается приток крови, повышается потоотделение).

С ростом напряжения сопротивление тела уменьшается в десятки раз. При больших напряжениях приближается к наименьшему пределу 300 Ом. В России в качестве расчетных значений сопротивление человека равно 1000 Ом при напряжении, приложенном к телу, равное 50 В и выше и сопротивление человека равное 6000 Ом при приложенном напряжении 36 В. Опыты показывают, что сопротивление тела человека постоянному току больше, чем переменному любой частоты. Разница в значениях сопротивлений постоянному и переменному (50 Гц) током особенно велико при малых напряжениях – до 10 В. С ростом приложенного напряжения эта разница уменьшается и начиная с 40-80 В сопротивление тела человека как постоянному, так и переменному току промышленной частоты становится практически одинаковым.

На значение сопротивления тела человека влияют и другие факторы, хотя в значительно меньшей степени. Пол и возраст. У женщин, как правило, сопротивление тела меньше, чем у мужчин, а у детей – меньше, чем у взрослых, у молодых людей меньше, чем у пожилых. Объясняется это, очевидно, тем, что у одних людей кожа тоньше и нежнее, у других — толще и грубее.

Физическое раздражение снижает сопротивление тела на 20-25%.

Повышенная температура окружающего воздуха (30-450 С) или тепловое облучение человека, вызывает некоторое понижение сопротивление тела.

Принцип работы трансформатора отбора напряжения в шкафу отбора напряжения
Обслуживание устройств АПВ
Назначение и область применения устройства автоматического повторного включения

Электрическое сопротивление человека. Сопротивление тела у людей. « ЭлектроХобби

Человеческое тело, как и любое другое тело живого организма, имеет свойство проводить через себя электрический ток. Разные живые ткани в организме имеют различную проводимость (сопротивление). К примеру — кожа, жировая ткань, кости – имеют большое сопротивление, а кровь, мышечная масса и особенно головной и спинной мозг – малое. Кожа имеет большое удельное электрическое сопротивление, что впоследствии и определяет фактическое сопротивление человеческого тела.

Кожа человека, как известно, имеет два слоя:

наружный слой кожи (также ещё называется эпидермис) состоит из несколько слоёв, самый верхний из которых называется роговым и представляет собой множество рядов отмерших и ороговевших клеток. В чистом и сухом виде этот слой можно характеризовать как диэлектрик (он имеет очень большое электрическое сопротивление). Следующий слой эпидермиса (носит название — ростковый) гораздо тоньше рогового и имеет значительно большую электрическую проводимость (меньшее сопротивление).

внутренний слой кожи (называется дерма) представляет собой живую ткань. Данный слой дермы имеет малое электрическое сопротивление.

Электрическое сопротивление обычного человека при условии, что кожа у него чистая, сухая и неповреждённая (измеренное напряжением 15-20 Вольт) лежит в пределах 3 — 100 кОм (1кОм = 1000 Ом), в некоторых случаях и более. Сопротивление тела человека, а именно проводимость между двух электродов, которые касаются поверхности кожи, можно рассматривать как 3 сопротивления включённых последовательно: наружные слои (эпидермиса) представляют собой первое сопротивление, и внутренние слои является вторым и третьим сопротивлением, включающим в себя сопротивления внутреннего слоя кожи и сопротивление внутренних тканей.

Наружное сопротивление человека обладает не только активным сопротивлением, а ещё и ёмкостным, поскольку в самом месте контактирования электродов с человеческим телом образовывается некое подобие конденсатора, в роли обкладок которого являются сами электроды и ткани тела человека, хорошо проводящие электрический ток, что находятся под наружным слоем кожи, ну, а диэлектриком (изолятором между обкладками) в данном случае будет выступать наружный слой кожи (эпидермис).

Ёмкостная составляющая, присутствующая в сопротивлении человека обуславливает влияние, как рода электрического тока, так и его частоты на общую величину сопротивления тела. При частоте 10 — 20 кГц и свыше можно утверждать, что поверхностный слой кожи почти полностью утрачивает своё сопротивление, и общее сопротивление человека в данном случае будет состоять лишь из внутреннего сопротивления тела (сопротивление дермы и внутренних тканей).

Общее состояние кожи в значительной мере оказывает влияние на величину электрического сопротивления человека. При повреждении рогового слоя кожи (царапины, порезы, ссадины и т.д.) происходит снижение сопротивления человека до величины, приближенного к значению внутреннего сопротивления, а это, естественно, повышает опасность поражения электрическим током. Подобное влияние может оказываться и в случае увлажнения кожи водой или потом.

При электрическом переменном токе промышленной частоты (50 герц) берут во внимание только активное сопротивление человека (его тела) и соотносят его с величиной равной 1 кОм. В действительности данное электрическое сопротивление есть величина непостоянная, что имеет нелинейную характеристику и зависит от дополнительных условий, в том числе от параметров электрической цепи, состояния кожи, состояния окружающей среды, физиологии человека и т.д.

Так как сопротивление кожи у одного и того же человека может быть неодинаковое в разных местах и частях тела, то, естественно, на его сопротивление сильно будет влиять конкретное место прикосновения электрических контактов, а также их общая площадь. Величина электрического тока и длительность воздействия на тело оказывают прямое влияние на полное сопротивление человека: с увеличением значения тока и времени его прохождения, сопротивление будет понижаться, потому что происходит местный нагрев участков кожи, а это, само собой, ведёт к расширению сосудов, тем самым усиливая снабжение данного участка тела кровью, увеличения его потоотделение. Увеличение напряжения, воздействующее на тело человека, вызывает понижение сопротивления кожи в 10-ки раз, следовательно, и общее сопротивление человека, снижается до предела 300 — 500 Ом. А это опасно.

P.S. Всякие случайности хороши в том случае, когда они имеют положительный характер. Случайный удар электрическим током нельзя отнести к таковым. Следовательно, будьте внимательны и осторожны при работе с электричеством.

Электрическое сопротивление человеческого тела: значение в омах

Сопротивление происходит от слова “сопротивляться”. В электронике  есть такое понятие, как Ом.  Что это такое и с чем его едят? Для более развернутого ответа, давайте рассмотрим вот такую схему:

  • Буквы в кружочках – это измерительные приборы

Вольтметр служит для измерения напряжения, а амперметр – для измерения силы тока. Как ими правильно пользоваться читаем в этой статье.

Итак, если пропустить по проводу электрический ток с силой тока в 1 Ампер, а на концах этого провода у нас появится напряжение в 1 Вольт,  это значит, что наш провод обладает сопротивлением в 1 Ом.

В электротехнике и электронике сопротивление обозначается буквой R. Например, тело человека имеет сопротивление от  нескольких сотен Ом и до 100 кОм. Для расчетов берут 1 кОм.

Это зависит от многих факторов, таких как пол, возраст, состояние кожи, сила прикосновения проводников к коже, уровень алкоголя в крови и тд.

Медный провод длиной в метр и сечением в  1 мм2  имеет сопротивление 0,1 Ом.

От чего зависит сопротивление

  1. Какой из предметов будет оказывать большее сопротивление электрическому току?
  2. Садовый шланг
  3. или нефтяная магистраль?

Конечно же садовый шланг. Почему? Да потому что его диаметр намного меньше, чем у нефтяной магистрали.

  • А теперь ответьте на такой вопрос, какой шланг будет обладать большим сопротивлением, с учетом того, что их длины и диаметры равны?
  • Гофрированный
  • или гладкий?

Разумеется гофрированный. Его стенки будут препятствовать потоку воды.

И еще один нюанс. У нас есть садовый гофрированный шланг. Мы обрезали от него небольшую длину, но все равно остался еще большой моток шланга

У какого шланга будет большее сопротивление потоку воды? Думаю, у того, который длиннее.

Формула сопротивления


Как ни странно, но дела с проводом обстоят точно также. Чем тоньше и длиннее провод, тем больше его сопротивление электрическому току.

Большую роль играет также материал, из которого он изготовлен.  Различные материалы по разному проводят электрический ток.

Есть те, которые замечательно проводят ток, типа серебра, а есть те, которые почти не пропускают через себя электрический ток, типа фарфора.

Поэтому, формула будет иметь такой вид:

В технике до сих пор применяется устаревшая единица измерения удельного сопротивления Ом х мм2 /м.  Чтобы перевести  в Ом х м, достаточно умножить на 10-6, так как 1 мм2=10-6м2.

Как вы видите из таблицы выше, самым маленьким удельным сопротивлением обладает серебро, поэтому провод из серебра будет наилучшим проводником в конструировании радиоэлектронных устройств. Ну а самым распространенными и дешевыми – медь и алюминий. Именно эти два металла в основном используются во всей электронной и электротехнической промышленности.

Вещества, которые оказывают наименьшее сопротивление электрическому току и обладают очень малым сопротивлением называются проводниками, а вещества, которые обладают ну очень большим сопротивлением электрическому току и почти его не пропускают через себя, называются диэлектриками. Между ними стоит класс полупроводников.

Резисторы

В электронике уже имеются специальные радиоэлектронные компоненты. Их называют резисторами.

  1. Существуют постоянные резисторы, у которых сопротивление практически не меняется:
  2. а есть также и переменные резисторы:
  3. С помощью них можно изменять сопротивление в каком-либо определенном диапазоне.

Последовательное и параллельное соединение резисторов

  • В электрических схемах постоянные резисторы обозначаются так:
  • переменные выглядят немного по-другому

Все вышеописанные резисторы можно соединять параллельно или последовательно. При параллельном соединении выводы резисторов соединятся в общих точках.

  1. В этом случае, чтобы узнать общее сопротивление всех резисторов в цепи, достаточно будет воспользоваться формулой, где значение между точками А и В (RAB) и есть то самое R общее:
  2. При последовательном соединении номиналы резисторов просто суммируются
  3. В этом случае

Резюме

Сопротивление играет главную роль в электронике и электротехнике. Любой материал во Вселенной обладает сопротивлением электрическому току. Некоторые материалы очень плохо пропускают через себя электрический ток, а некоторые материалы, такие как серебро и медь, обладают очень малым сопротивлением и отлично пропускают через себя электрический ток.

На сопротивление влияют также такие параметры, как материал, площадь поперечного сечения материала, а также его длина. Материалы, которые отлично проводят через себя электрический ток называются проводниками, а которые препятствую протеканию электрического тока – диэлектриками.

Резисторы – специальные радиоэлементы в электронике, которые обладают определенным номиналом сопротивления и выполняют различные функции.

Источник: https://www.RusElectronic.com/soprotivljenije/

3. И снова Закон Ома!

И снова Закон Ома!

Довольно часто нам приходится слышать такие фразы, как «Ударило током» или «Убило током», и ни какого упоминания о напряжении. Исходя из этого, у вас может сложиться впечатление, что для человека опасен ток, а не напряжение.

Какой-то элемент истины здесь имеет место быть.

Однако, если напряжение не представляет никакой опасности, то зачем пишутся предупреждающие таблички примерно такого содержания: «ОСТОРОЖНО — ВЫСОКОЕ НАПРЯЖЕНИЕ!»?

По большому счету принцип «опасности тока» верен. Электрический ток вызывает ожоги тканей тела, блокирует мышцы и останавливает сердце, но он не может возникнуть сам по себе. Чтобы создать поток электронов через тело человека, к нему нужно приложить напряжение. При выполнении расчетов мы так же должны учесть сопротивление, которое тело человека оказывает электрическому току.

Если с помощью Закона Ома мы выразим силу тока через известные значения напряжения и сопротивления, то получим следующее уравнение:

Сила тока, проходящего через тело человека прямопропорциональна величине напряжения, приложенного к двум его точкам, и обратнопропрорциональна сопротивлению между этими точками. Очевидно, чем больше величина напряжения, создающего поток электронов, тем легче эти электроны будут проходить через конкретную величину сопротивления.

Следовательно, высокое напряжение опасно для жизни, потому что оно создает большой ток, который может травмировать или убить человека. И наоборот, чем большее сопротивление оказывает тело электрическому току, тем медленнее будут течь через него электроны при заданной величине напряжения.

Проще говоря, опасность того или иного напряжения зависит от величины сопротивления, оказываемого телом человека потоку электронов.

Сопротивление тела не является фиксированной величиной. Оно изменяется от человека к человеку, и время от времени. На измерении электрического сопротивления между пальцами рук и ног основывается метод определения процентного содержания жира в организме.

Разные проценты содержания жира обеспечивают разные сопротивления, и это только одна из величин, влияющая на электрическое сопротивление тела человека.

Чтобы метод работал точно, человек за несколько часов до теста должен регулировать потребление жидкости, а это говорит о том, что гидратация является еще одним фактором, влияющим на сопротивление человеческого тела.

Сопротивление так же зависит от того, между какими частями тела мы его будем измерять: между руками, между ногами, между рукой и ногой и т.д.

Необходимо учесть и тот фактор, что прекрасными проводниками электричества являются пот, богатый солями и минералами, а также кровь, с ее высоким содержанием проводящих химических элементов.

 Таким образом, контакт между проводом и потными руками или руками с кровоточащей раной будет обладать гораздо меньшим сопротивлением, чем контакт между проводом и руками с сухой, чистой кожей.

Измеряя сопротивление своего тела чувствительным измерительным прибором, путем сжимания его щупов пальцами рук,  я получил значение 1 миллион Ом (1 МОм).

При этом прибор показывает меньшее сопротивление, когда я плотно сжимаю щупы, и большее сопротивление — когда я ослабляю пальцы. Руки мои при этом чисты и сухи.

Если бы я работал во влажной и грязной производственной среде, то сопротивление между моими руками было бы намного меньше, представляя большую угрозу поражения электрическим током.

Итак, какая же величина тока опасна для человека?. Ответ на этот вопрос зависит от нескольких факторов. Значительное влияние на то, как электрический ток воздействует на человека, оказывает химический состав его тела.

Некоторые люди очень чувствительны к току, и поэтому испытывают непроизвольное сокращение мышц даже от разряда статического электричества, который другие люди могут и не почувствовать.

Несмотря на эти различия, посредством тестов были выведены примерные значения тока (очень небольшие), которые могут оказать вредное воздействие на организм человека. Все значения в таблице даны в миллиамперах (миллиампер равен 1/1000 ампера):

«Гц» является сокращенным обозначением единицы измерения Герц, которая служит мерой скорости чередования направлений переменного тока. Эта мера иначе известна как частота.

Так, заголовок «60 Гц АС» одного из столбцов таблицы означает что все значения этого столбца относятся к переменному току, который чередуется с частотой 60 циклов в секунду (1 цикл равен периоду времени, в течении которого поток электронов сначала движется в одном направлении, а потом в другом).

Последняя колонка, с надписью «10 кГц АС», относится к переменному току, который совершает десять тысяч циклов в секунду.

Следует иметь ввиду, что все вышеприведенные цифры являются приблизительными, поскольку реакция на ток  людей с разным химическим составом тела будет различной. Существует предположение, что достаточно пропустить переменный ток величиной в 17 миллиампер через грудь человека, чтобы при определенных условиях вызвать у него аритмию сердца.

Большинство данных таблицы, касающихся аритмии сердца, взяты из опытов над животными. И это естественно, ведь никто не будет проводить такие эксперименты на людях, в связи с чем имеющиеся данные весьма приблизительны.

Если вас интересует вопрос, почему женщины более восприимчивы к электрическому току чем мужчины, то здесь мы вам не поможем — для нас это тоже загадка.

Теперь давайте предположим, что я взялся сухими и чистыми руками за контакты источника напряжения переменного тока частотой 60 Гц. Какое напряжение должно быть у этого источника, чтобы создать ток величиной 20 миллиампер (при таком токе я не смогу самостоятельно отпустить контакты источника)? Ответ на этот вопрос можно найти в Законе Ома (U = IR):

  • U = IR
  • U = (20 мA)(1 МОм)
  • U = 20,000 вольт, или 20 кВ

Имейте в виду, что это сценарий «лучшего случая» с точки зрения электробезопасности (чистая, сухая кожа), а полученная величина напряжения, с огромной долей вероятности, вызовет оцепенение человека.

Гораздо меньшее напряжение потребуется для вызова болевых ощущений.

Следует учесть так же и тот момент, что физиологические эффекты воздействия различных токов на разных людей могут значительно отличаться, поэтому наши расчеты являются только приблизительной оценкой действительности.

Если я смочу пальцы своих рук водой, имитируя пот, то сопротивление моего тела между руками составит всего 17000 Ом (17 кОм). Обратите внимание, что в нашем случае с тонкими металлическими щупами измерительного прибора контактирует по одному пальцу каждой руки. Повторно вычислив напряжение, необходимое для получения тока величиной 20 мА, мы получим следующее значение:

  1. U = IR
  2. U = (20 мA)(17 kОм)
  3. U = 340 вольт

В этом случае достаточно напряжения 340 вольт, чтобы создать ток 20 миллиампер через тело человека. Однако, смертельный удар током можно получить и от меньшего напряжения если увеличить площадь контакта, уменьшив тем самым его сопротивление.

Примером такого контакта служит кольцо на пальце (золото обернутое вокруг пальца создает превосходный контакт для поражения электрическим током) или большой металлический предмет, такой как труба или ручка инструмента.

Сопротивление организма при этом понизится до 1000 Ом (10 кОм), что создаст реальную угрозу поражения низкими значениями напряжения:

  • U = IR
  • U = (20 мA)(1 kОм)
  • U = 20 вольт

Таким образом, чтобы создать ток величиной 20 мА и вызвать оцепенение человека, достаточно напряжения 20 вольт. Ранее мы упомянули предположение, что сила тока 17 мА, пропущенная через грудь человека, при определенных условиях может вызвать аритмию сердца. Так вот, если сопротивление между руками человека будет равно 1 кОм, то для создания этого опасного условия потребуется всего 17 вольт:

  1. U = IR
  2. U = (17 мA)(1 kОм)
  3. U = 17 вольт

В этих расчетах мы показали вам «наихудший» сценарий для напряжения переменного тока частотой 60 Гц и отличной проводимости человеческого тела. Данный пример дает наглядную картину опасности даже небольших значений напряжения.

Понизить сопротивление человеческого тела до 1000 Ом можно не только путем воздействия рассмотренных выше экстремальных факторов (например плотным контактом золотого кольца с пальцем).

Оно может уменьшиться при длительном воздействии напряжения (например, когда человек под действием тока не может разжать руку, и только крепче сжимает проводник).

Одновременно с уменьшением сопротивления увеличивается сила тока при фиксированном напряжении.

Ниже приведены примерные значения сопротивлений точек контакта человека с различными предметами в различных условиях:

  • Контакт пальца с проводом: от 40 000 Ом до 1 000 000 Ом (сухой палец), от 4 000 Ом до 15 000 Ом (влажный палец).
  • Контакт руки с проводом: от 15 000 Ом до 50 000 Ом (сухая рука), от 3 000 Ом до 5 000 Ом (влажная рука).
  • Контакт руки с металлическими плоскогубцами: от 5 000 Ом до 10 000 Ом (сухая рука), от 1 000 Ом до 3 000 Ом (влажная рука).
  • Контакт с ладонью: от 3 000 Ом до 8 000 Ом (сухая ладонь), от 1 000 Ом до 2 000 Ом (влажная ладонь).
  • Контакт (обхват) одной руки с 1,5-дюймовой металлической трубой: от 1 000 Ом до 3 000 Ом (сухая рука), от 500 Ом до 1 500 Ом (влажная рука).
  • Контакт (обхват) двух рук с 1,5-дюймовой металлической трубой: от 500 Ом до 1 500 Ом (сухие руки), от 250 Ом до 750 Ом (влажные руки).
  • Контакт руки, погруженной в проводящую жидкость, с этой жидкостью: от 200 Ом до 500 Ом.
  • Контакт ноги, погруженной в проводящую жидкость, с этой жидкостью: от 100 Ом до 300 Ом.         

Обратите внимание на значения сопротивлений в двух случаях с 1,5-дюймовой металлической трубой. Если трубу обхватить двумя руками, то сопротивление будет ровно в два раза меньше, чем при обхвате этой же трубы одной рукой.

Две руки, сжимающие металлическую трубу, увеличивают площадь контакта в два раза по сравнению с одной рукой. Это очень важное обстоятельство: электрическое сопротивление между любыми контактирующими объектами уменьшается с увеличением площади контакта при прочих равных условиях. В этом случае электроны текут из трубы в тело (или наоборот) по двум параллельным маршрутам.

Как вы увидите позже, общее сопротивление параллельной цепи всегда меньше (или равно) любого из сопротивлений этой цепи.

В промышленности пороговым значением опасного напряжения считается, как правило, напряжение величиной 30 вольт. Осторожный человек должен рассматривать любое напряжение, превышающее это значение, как опасное.

Работая с электричеством он должен содержать свои руки чистыми и сухими, а так же снять все металлические украшения, которые представляют опасность и при более низких значениях напряжения.

Такие украшения, выступая в качестве контактов между двумя точками цепи, способны провести достаточный ток, чтобы сжечь кожу.

Опасными могут быть и напряжения менее 30 вольт, если они достаточны чтобы вызвать неприятные ощущения, в результате которых человек может совершить резкое движение и вступить в контакт с другим, более высоким напряжением или иным источником повышенной опасности.

Автор статьи вспоминает, как однажды он ремонтировал свой автомобиль в жаркий летний день. По причине теплой погоды он был в шортах, и работая с аккумуляторной батареей прислонился оголенной частью ног к хромированному бамперу машины.

Когда он коснулся металлическим ключом положительного контакта 12-вольтовой батареи, то почувствовал покалывание в точке контакта ноги с бампером.

Таким образом, надежный контакт с металлом и потная кожа позволили почувствовать удар током  от электрического потенциала значением всего-лишь 12 вольт.

К счастью, в этом случае ничего плохого не случилось.

Но, если бы двигатель автомобиля был запущен, и воздействие тока почувствовала рука а не нога, то автор, возможно, рефлексивно дернул бы ее в сторону вращающегося вентилятора или уронил бы ключ на клеммы аккумулятора (вызвав тем самым короткое замыкание и сноп искр). Этот пример иллюстрирует еще один важный урок электробезопасности — электрический ток может послужить косвенной причиной травмирования.

Очень важное значение имеет путь, по которому ток течет через тело человека.

Благодаря тому, что электрический ток оказывает влияние на все мышцы организма находящиеся на его пути, в том числе и на такие жизненно-важные, как сердце и легкие, наиболее опасным будет такой ток, который проходит через грудь человека. Это сценарий возможен в том случае, если человек соприкоснется с источником напряжения двумя руками.

В целях недопущения такого сценария, при работе со схемой (находящейся под напряжением) желательно использовать только одну руку, засунув вторую при этом в карман (чтобы случайно ничего ей не тронуть).

Конечно, безопаснее было бы работать с обесточенной схемой, но на практике это не всегда возможно. Если схема находится под напряжением, то работать с ней предпочтительнее правой рукой. А почему правой, спросите вы.

Во-первых, если человек правша (каких большинство), то ему удобнее будет работать именно этой рукой, а во-вторых — сердце расположено в левой части грудной клетки.

Лучшей защитой от удара электрическим током является сопротивление, которое может быть добавлено к телу при помощи изолированных инструментов, перчаток, сапог и других приспособлений.

Как вы уже знаете, ток в цепи равен напряжению деленному на общее сопротивление потоку электронов.

Наибольший эффект сопротивления будут иметь при расположении их таким образом, чтобы создать только один путь для потока электронов (подробнее на этом мы остановимся в последующих статьях):

Ниже представлена эквивалентная схема человека, экипированного перчатками и сапогами:

В этом случае суммарное (общее) сопротивление сапог, тела и перчаток потоку электронов будет больше, чем сопротивление каждого из компонентов по отдельности.

Безопасность является одной из причин, по которой электрические провода покрываются пластмассовой или резиновой изоляцией, которая значительно увеличивает сопротивление между проводником и прикоснувшемуся к нему человеком. Однако покрывать изоляцией высоковольтные провода линий электропередач слишком дорого, поэтому безопасность в этом случае достигается путем подвешивания их на столбы высоко над землей.

Краткий обзор:

  • Электрический ток оказывает вредное воздействие на организм человека. Чем больше напряжение, тем больший и опасный ток оно производит. Уменьшить силу тока может сопротивление. Высокое сопротивление является хорошей защитой от удара электрическим током.
  • Напряжение величиной выше 30 вольт способно создать опасный ток.
  • Не нужно надевать металлические украшения при работе с электрическими схемами. Кольца, ремешки часов, ожерелья, браслеты и другие подобные вещи обеспечивают отличный электрический контакт с телом человека и способны провести достаточный ток, чтобы вызвать ожог кожи даже при низких напряжениях.
  • Опасность несет и такое низкое напряжение, которое непосредственно не может поразить человека. Его может быть достаточно, чтобы человек отдернул руку и вступил в контакт с другим, находящимся вблизи источником опасности.
  • В целях предотвращения прохождения опасного тока через грудь человека, работать с запитанной схемой необходимо одной рукой (по возможности — правой).  

Источник: http://www.radiomexanik.spb.ru/3.-elektrobezopasnost/3.-i-snova-zakon-oma.html

Электрическое сопротивление человека. Сопротивление тела у людей

Тема: какое электрическое сопротивление имеет человеческое тело

Кожа человека, как известно, имеет два слоя:

наружный слой кожи (также ещё называется эпидермис) состоит из несколько слоёв, самый верхний из которых называется роговым и представляет собой множество рядов отмерших и ороговевших клеток.

В чистом и сухом виде этот слой можно характеризовать как диэлектрик (он имеет очень большое электрическое сопротивление).

Следующий слой эпидермиса (носит название — ростковый) гораздо тоньше рогового и имеет значительно большую электрическую проводимость (меньшее сопротивление).

внутренний слой кожи (называется дерма) представляет собой живую ткань. Данный слой дермы имеет малое электрическое сопротивление.

Электрическое сопротивление обычного человека при условии, что кожа у него чистая, сухая и неповреждённая (измеренное напряжением 15-20 Вольт) лежит в пределах 3 — 100 кОм (1кОм = 1000 Ом), в некоторых случаях и более.

Сопротивление тела человека, а именно проводимость между двух электродов, которые касаются поверхности кожи, можно рассматривать как 3 сопротивления включённых последовательно: наружные слои (эпидермиса) представляют собой первое сопротивление, и внутренние слои является вторым и третьим сопротивлением, включающим в себя сопротивления внутреннего слоя кожи и сопротивление внутренних тканей.

Ёмкостная составляющая, присутствующая в сопротивлении человека обуславливает влияние, как рода электрического тока, так и его частоты на общую величину сопротивления тела.

При частоте 10 — 20 кГц и свыше можно утверждать, что поверхностный слой кожи почти полностью утрачивает своё сопротивление, и общее сопротивление человека в данном случае будет состоять лишь из внутреннего сопротивления тела (сопротивление дермы и внутренних тканей).

Общее состояние кожи в значительной мере оказывает влияние на величину электрического сопротивления человека. При повреждении рогового слоя кожи (царапины, порезы, ссадины и т.д.

) происходит снижение сопротивления человека до величины, приближенного к значению внутреннего сопротивления, а это, естественно, повышает опасность поражения электрическим током.

Подобное влияние может оказываться и в случае увлажнения кожи водой или потом.

При электрическом переменном токе промышленной частоты (50 герц) берут во внимание только активное сопротивление человека (его тела) и соотносят его с величиной равной 1 кОм.

В действительности данное электрическое сопротивление есть величина непостоянная, что имеет нелинейную характеристику и зависит от дополнительных условий, в том числе от параметров электрической цепи, состояния кожи, состояния окружающей среды, физиологии человека и т.д.

P.S. Всякие случайности хороши в том случае, когда они имеют положительный характер. Случайный удар электрическим током нельзя отнести к таковым. Следовательно, будьте внимательны и осторожны при работе с электричеством.

Источник: https://electrohobby.ru/elektr-soprot-chelov-soprot-tel.html

Сопротивление тела человека принимается равным 1000 Ом

  • МЕТОДИЧЕСКИЕ УКАЗАНИЯ
  • по выполнению лабораторных работ по курсу
  • «Безопасность жизнедеятельности»
  • для студентов всех специальностей
  • ОЦЕНКА ЭФФЕКТИВНОСТИ СПОСОБОВ ЗАЩИТЫ ЧЕЛОВЕКА ОТ ПОРАЖЕНИЯ ЭЛЕКТРИЧЕСКИМ ТОКОМ
  • Красноярск, 2012 г.
  • УДК

Рецензент: Калинин А.А., к.т.н., профессор, академик МАН экологии и безопасности.

Оценка эффективности способов защиты человека от поражения электрическим током. Методические указания по выполнению лабораторных работ для студентов всех специальностей/ составили Емец А.А., Красноярск, 2012, 19 с.

  1. Печатается по решению
  2. Редакционно-издательского совета университета
  3. Ó СФУ, 2012
  4. Печатается в авторской редакции
  5. Лабораторные работы
  6. « Оценка эффективности способов защиты человека от поражения электрическим током»
  7. Цель работы— изучение количественных и качественных характеристик защитного заземления, защитного зануления и устройства защитного отключения (УЗО), как средств защиты человека от поражения электрическим током.
  8. ОБЩИЕ СВЕДЕНИЯ
  9. ЭЛЕКТРОБЕЗОПАСНОСТЬ
  10. Электробезопасность — система организационных и технических мероприятий и средств, обеспечивающих защиту работающих от воздействия электрического тока.
  11. Поражение человека электрическим током может произойти при прикосновениях:
  12. — к токоведущим частям, находящимся под напряжением;
  13. — отключенным токоведущим частям, на которых остался заряд или появилось напряжение в результате случайного включения;
  14. — к металлическим нетоковедущим частям электроустановок после перехода на них напряжения с токоведущих частей.
  15. Характер и последствия поражения человека электрическим током зависят от ряда факторов, в том числе и от:
  16. — электрического сопротивления тела человека,
  17. — величины и длительности протекания через него тока,
  18. — рода и частоты тока,
  19. — схемы включения человека в электрическую цепь,
  20. — состояния окружающей среды
  21. — индивидуальных особенностей организма.
  • Электрическое сопротивление тела человека складывается из сопро­тивления кожи и сопротивления внутренних тканей.
  • При расчетах
  • сопротивление тела человека принимается равным 1000 Ом.
  • Сила тока, протекающего через тело человека, является главным фактором, от которого зависит исход поражения: чем больше сила тока, тем опаснее последствия.

Сила тока, протекающего через тело человек, при которой человек начинает ощущать проходящий через него ток, называется пороговым ощутимым током. Обычно, при частоте токапромышленной частоты 50 Гц пороговым ощутимым является значение 0,5…1,5 мА.

Ток силой 10… 15 мА вызывает сильные и непроизвольные судороги мышц, которые человек не в состоянии преодолеть, т. е. он не может разжать руку, которой касается токоведущей части, отбросить от себя провод, оказываясь как бы прикованным к токоведущей части. Такой ток называется пороговым неотпускающим.

При силе тока 20…25 мА у человека происходит судорожное сокращение мышц грудной клетки, затрудняется и даже прекращается дыхание, что может привести к смерти вследствие прекращения работы легких.

Ток силой 100 мА является смертельно опасным, так как он в этом случае оказывает непосредственное влияние на мышцы сердца, вызывая его остановку или фибрилляцию (быстрые хаотические и разновременные сокращения волокон сердечной мышцы), при которой сердце перестает работать.

Длительность протекания тока через тело человека определяет исход поражения им, так как с течением времени резко возрастает сила тока вследствие уменьшения сопротивления тела, и также потому, что в организме человека накапливаются отрицательные последствия воздействия тока.

  1. Опасность электрических сетей.
  2. На производстве используются следующие виды электрических сетей:
  3. — трехфазные электрические сети с изолированной нейтралью;
  4. — трехфазные электрические сети с заземленной нейтралью;
  5. — однофазные электрические сети.
  6. Опасность трехфазных электрических сетей с изолированнойнейтралью.

Провода электрических сетей по отношению к земле имеют емкость и активное сопротивление — сопротивление утечки, равное сумме сопротивлений изоляции и пути тока на землю (рис 5.10).

Для упрощения анализа можно принять их равными, т. е.

Са=Св= Сс и rА = rВ = rС = r

При прикосновении человека к одному из фазных проводов (однофазное сопротивление) исправной сети проводимость этого провода относительно земли уменьшается и происходит смещение нейтрали. Ток через человека в этом случае выражается зависимостью:

где — фазное напряжение сети; — сопротивление цепи человека (Rч = rтч + rод + rоб + rоп), rтч — сопротивление тела человека; rод— сопротивление одежды 0,5…1 кОм для влажной ткани и 10…15 кОм — для сухой; rоб

Источник: https://megaobuchalka. ru/10/3826.html

Чему равно сопротивление тела человека | Режимщик

Электрическое сопротивление различных тканей тела человека не одинаково.

Например, при токе частотой 50 Гц удельное сопротивление равно: кости – 107 Ом∙м, кожа сухая – 105 Ом∙м, крови – 1,7 Ом∙м.

При сухой, чистой и неповрежденной коже сопротивление тела, измеренное, при напряжении 15-20 В переменного тока (50 Гц), колеблется в пределах от 1 до 10 кОм, а иногда и в более широких пределах.

Сопротивление кожи, а следовательно сопротивление тела в целом резко уменьшается при повреждении ее рогового слоя, наличие влаги на ее поверхности, интенсивном потовыделении и загрязнении.

Электрическое сопротивление тела человека зависит так же от места приложения электродов к телу, значений тока, проходящего через человека, и приложенного к телу напряжения, рода и частоты тока, площади электродов, длительности прохождения тока через человека и некоторых других факторов.

Увеличение тока приводит к снижению сопротивления соответствующих участков кожи, за счет местного нагрева кожи и действия на центральную нервную систему (усиливается приток крови, повышается потоотделение). С ростом напряжения сопротивление тела уменьшается в десятки раз.

При больших напряжениях приближается к наименьшему пределу 300 Ом. В России в качестве расчетных значений сопротивление человека равно 1000 Ом при напряжении, приложенном к телу, равное 50 В и выше и сопротивление человека равное 6000 Ом при приложенном напряжении 36 В.

Опыты показывают, что сопротивление тела человека постоянному току больше, чем переменному любой частоты. Разница в значениях сопротивлений постоянному и переменному (50 Гц) током особенно велико при малых напряжениях – до 10 В.

С ростом приложенного напряжения эта разница уменьшается и начиная с 40-80 В сопротивление тела человека как постоянному, так и переменному току промышленной частоты становится практически одинаковым.

На значение сопротивления тела человека влияют и другие факторы, хотя в значительно меньшей степени. Пол и возраст. У женщин, как правило, сопротивление тела меньше, чем у мужчин, а у детей – меньше, чем у взрослых, у молодых людей меньше, чем у пожилых. Объясняется это, очевидно, тем, что у одних людей кожа тоньше и нежнее, у других — толще и грубее.

Физическое раздражение снижает сопротивление тела на 20-25%.

Повышенная температура окружающего воздуха (30-450 С) или тепловое облучение человека, вызывает некоторое понижение сопротивление тела.

Принцип работы трансформатора отбора напряжения в шкафу отбора напряжения Обслуживание устройств АПВ Назначение и область применения устройства автоматического повторного включения

Источник: http://elektro-rezhim.ru/soprotivlenie-tela-cheloveka/

25) Электрическое сопротивление тела человека. Электрическое сопротивление тела человека

Тело
человека является проводником
электрического тока. Различные ткани
тела оказывают току разное сопротивление:
кожа, кости, жировая ткань — большое, а
мышечная ткань, кровь и особенно спинной
и головной мозг — малое. Кожа обладает
очень большим удельным сопротивлением,
что является главным фактором, определяющим
сопротивление всего тела человека.

Кожа
состоит из двух основных слоёв: наружного,
называемого эпидермисом, и внутреннего,
являющегося собственно кожей и носящего
название дермы. Наружный слой кожи —
эпидермис, в своё очередь имеет несколько
слоёв, из которых самый верхний называется
роговым и состоит из многих рядов
ороговевших клеток.

В
сухом и незагрязнённом виде роговой
слой можно рассматривать как диэлектрик.
Другие слои эпидермиса (ростковый слой)
в несколько раз тоньше рогового слоя и
обладает значительно меньшим
сопротивлением.

Внутренний
слой кожи — дерма является живой тканью.
Электрическое сопротивление дермы
невелико.

Сопротивление
тела человека при сухой, чистой и
неповреждённой коже (измеренное при
напряжении до 15-20 В) колеблется в пределах
примерно от 3000 до 100 000 Ом, а иногда и
более.

Сопротивление тела человека, то
есть сопротивление между двумя
электродами, наложенными на поверхность
тела, можно условно считать состоящим
из трёх последовательно включённых
сопротивлений: двух одинаковых наружных
слоя кожи (эпидермиса), составляющих в
совокупности так называемое наружное
сопротивление тела человека, и одного,
называемого внутренним сопротивлением
тела, включающим в себя два сопротивления
внутреннего слоя кожи (дермы) и
сопротивление внутренних тканей тела.

Наружное
сопротивление тела обладает не только
активным сопротивлением, но и ёмкостным,
так как в месте прикосновения электродов
к телу человека образуются как бы
конденсаторы, обкладками которых
являются электроды и хорошо проводящие
токи ткани тела человека, лежащие под
наружным слоем кожи, а диэлектриком —
наружный слой (эпидермис). Внутреннее
сопротивление тела считается чисто
активным.

Обычно
при переменном токе промышленной частоты
учитывают лишь активное сопротивление
тела человека и принимают его равным
1000 Ом. В действительности это сопротивление
— величина переменная, имеющая нелинейную
зависимость от множества факторов, в
том числе от состояния кожи, параметров
электрической цепи, физиологических
факторов и состояния окружающей среды.

Состояние
кожи

— очень сильно сказывается на величине
сопротивления тела человека.

Так,
повреждение рогового слоя, в том числе
порезы, царапины, ссадины и другие
микротравмы, могут снизить полное
сопротивление тела до значения, близкого
к величине внутреннего сопротивления,
что безусловно увеличивает опасность
поражения человека током. Такое же
влияние оказывает и увлажнение кожи
водой или за счёт пота, а также загрязнение
кожи проводящей пылью или грязью.

Поскольку
у одного итого же человека сопротивление
кожи неодинаково на разных участках
тела, то на сопротивление в целом
сказывается место приложения контактов,
а также их площадь.

Величина тока и
длительность его прохождения через
тело оказывают непосредственное влияние
на полное сопротивление: с ростом тока
и времени его прохождения сопротивление
падает, поскольку при этом усиливается
местный нагрев кожи, что приводит к
расширению её сосудов, а следовательно
к усилению снабжения этого участка
кровью и увеличению потовыделения.

Повышение
напряжения, приложенного к телу человека,
вызывает уменьшение в десятки раз
сопротивления кожи, а следовательно, и
полного сопротивления тела человека,
приближающегося в пределе к своему
наименьшему значению — 300-500 Ом.

Наличие
ёмкостной составляющей в сопротивлении
тела человека обусловливает влияние
рода и частоты тока на величину полного
сопротивления.

Так, при частоте 10-20 кГц
и более можно считать, что наружный слой
кожи практически утрачивает сопротивление
электрическому току, и полное сопротивление
кожи состоит только из внутреннего
сопротивления тела человека (то есть
из сопротивлений дермы и внутренних
тканей тела).

Источник: https://studfile.net/preview/5301451/page:16/

Сколько в Омах сопротивление у человеческого тела?

Войти через mail.ru
Виктор

  • 500 Ом, если без кожи.
  • Расчётное 1 кОм, практически — до 1 МОм.

Илья

Уточнить ты не сможешь, ибо человек не конкретный металл, сопротивление кожи всегда меняется. Зависить от возраста, от влажности, от настроение, от температуры тело. Вообщем начинается от 100 Ом, до 10 МОм

Андрей

Сопротивл ение у каждого чел овека разное и зависит от состояния чел овека. В среднем по правил ам расчета по ел ектробезопасности 1000 Ом.

Обычно 10кОм -максимум 500 кОм. Расчетное сопротивление для человека берется 1 кОм. Оно может быть у пьяного или сильно переживающего человека, с мокрыми руками, сильно потеющего, поврежденной кожей. Сопротивление имеет как активную составляющую, так и реактивную. Ток лентяй и течет где ему легче.

Если вы взяли 2 щупа в руки то он покажет сопротивление между рукой-плечами- 2 рукой. Ток через ноги не потечет в этом случае. Так что работай с током обычно только правой рукой, даже если ты левша. И не лезь к высокому напряжению. Это обычно считают более 42 В опасно для жизни и сила тока более 0,1 А.

Eugene

Оно разное для различных участков тела. В среднем при расчете электробезопасности его принимают равным 1 кОм. Самым большим сопротивлением обладает верхний слой кожи (3..

20 кОм) Сопротивление человека Для расчёта величины силы тока, протекающего через человека при попадании его под электрическое напряжение частотой 50 Гц, сопротивление тела человека условно принимается равным 1 кОм [5]. Эта величина имеет малое отношение к реальному сопротивлению человеческого тела.

В реальности сопротивление человека не является омическим, так как эта величина, во-первых, нелинейна по отношению к приложенному напряжению, во-вторых меняется во времени, в третьих, гораздо меньше у человека, который волнуется и, следовательно, потеет и т. д.

Серьёзные поражения тканей человека наблюдаются обычно при прохождении тока силой около 100 мА. Совершенно безопасным считается ток силой до 1 мА.

Удельное сопротивление тела человека весьма значительно (около 15 кОм) . Поэтому опасные токи могут быть достигнуты только при значительном напряжении.

Однако при наличии сырости сопротивление тела человека резко снижается и безопасным может считаться напряжение только ниже 12 В.

Другие вопросы из категории «Наука, Техника, Языки»

Источник: https://sprashivalka.com/tqa/q/18027055

Большая Энциклопедия Нефти и Газа

Cтраница 2

Электрическое сопротивление тела человека может меняться РІ широких пределах — РѕС‚ 100000 РґРѕ 300 РћРј.

Оно зависит от приложенного напряжения, длительности, прохождения тока и др.

Нервные, сердечные Рё легочные заболевания, влажность кожи также понижают электрическую сопротивляемость тела человека.  [16]

Электрическое сопротивление тела человека изменяется от 800 до 100000 Ом.

РћРЅРѕ зависит РѕС‚ РјРЅРѕРіРёС… факторов: состояния Р·РґРѕСЂРѕРІСЊСЏ, нервной системы, психического состояния — влажности кожи, состояния одежды, РѕР±СѓРІРё Рё РґСЂСѓРіРёС… причин.  [17]

Электрическое сопротивление тела человека складывается из сопротивления кожи и сопротивления внутренних тканей.

 [18]

Электрическое сопротивление тела человека изменяется РІ широком диапазоне-РѕС‚ 400 РґРѕ 100000 РћРј. Р—Р° опасную силу тока РЅР° основании последних исследований принимают 0 01 Рђ.  [19]

Электрическое сопротивление тела человека изменяется от 800 до 100000 Ом.

РћРЅРѕ зависит РѕС‚ РјРЅРѕРіРёС… факторов: состояния Р·РґРѕСЂРѕРІСЊСЏ, нервной системы, психического состояния, влажности кожи, состояния одежды, РѕР±СѓРІРё Рё РґСЂСѓРіРёС… причин.  [20]

Электрическое сопротивление тела человека РїСЂРё СЃСѓС…РѕР№ неповрежденной коже находится РІ пределах 10000 — 100 000 РѕРј.

Величина этого сопротивления определяется в основном сопротивлением внешнего рогового слоя кожи и при влажной коже может снизиться до тысяч ом.

РџСЂРё снятом СЂРѕРіРѕРІРѕРј слое кожи сопротивление уменьшается РґРѕ 800 — 1000 РѕРј.  [21]

Электрическое сопротивление тела человека складывается из сопротивления кожи и сопротивления внутренних тканей.

Кожа, РІ РѕСЃРЅРѕРІРЅРѕРј верхний ее слой толщиной 0 2 РјРј, состоящий РёР· мертвых ороговевших клеток, обладает большим сопротивлением, которое определяет общее сопротивление тела человека.  [22]

Электрическое сопротивление тела человека нелинейно.

Эта нелинейность чрезвычайно сложна: как уже указывалось, она зависит от физических, биофизических и биохимических факторов и о ней мало что достоверно известно.

Поэтому приходится делать СЂСЏРґ допущений Рё прежде всего считать РїСЂРё первом рассмотрении, что электрическое сопротивление тела человека линейно, Р° затем уже РїРѕ мере накопления экспериментального материала вносить необходимые уточнения. Основанием для такого допущения служит то, что большая часть биохимических Рё биофизических процессов проявляется РЅРµ РІ первое же мгновение после электротравмы, Р° РїРѕ истечении некоторого времени. РћР± этом свидетельствуют хотя Р±С‹ кривые СЂРёСЃ. 6 — 6, показывающие изменение электрического сопротивления тела человека РїРѕРґ влиянием физических раздражителей. Р�Р· этих кривых РІРёРґРЅРѕ, что изменение наступает РІ течение РјРёРЅСѓС‚, тогда как РїСЂРё оценке электрической цепи РІ процессе расследования несчастных случаев приходится решать РІРѕРїСЂРѕСЃ Рѕ переходных процессах, длительность которых РЅРµ превышает миллисекунд.  [23]

Электрическое сопротивление тела человека складывается из сопротивления кожи и сопротивления внутренних тканей.

 [24]

Величина электрического сопротивления тела человека изменяется от 800 до 100 000 Ом.

РћРЅР° зависит РѕС‚ РјРЅРѕРіРёС… факторов: состояния Р·РґРѕСЂРѕРІСЊСЏ, нервной системы, психического состояния, влажности кожи, состояния одежды, РѕР±СѓРІРё Рё РґСЂСѓРіРёС… причин.  [25]

�зучение электрического сопротивления тела человека сильно продвинулось вперед, когда в исследовательский обиход были внедрены измерения сопротивлений на высоких частотах.

Эти измерения впервые показали, что электрическое сопротивление внутренних органов является преимущественно активным сопротивлением, Р° емкостная составляющая полного сопротивления обусловлена РІ РѕСЃРЅРѕРІРЅРѕРј сопротивлением кожи.  [26]

�змерения электрического сопротивления тела человека обычно проводятся обстоятельно, но противоречия не сглаживаются.

Называется общее значение сопротивления в омах, но не указываются ни площади поверхностей электродов, ни силы, с которыми они давили на кожу.

Если сообщается Рѕ сопротивлении РїСЂРё петле СЂСѓРєР° — СЂСѓРєР°, то РЅРµ уточняется, РІ какой части СЂСѓРєРё — тыльной или ладонной — произошел контакт.  [27]

Зависимость электрического сопротивления тела человека Рё тока, проходящего через него, РѕС‚ величины приложенного напряжения показаны РЅР° СЂРёСЃ. 26, РіРґРµ 1 Рё 2 — переменный ток СЃ частотой 50 Гц; 3 Рё 4 — постоянный ток.  [28]

Р�зучение электрического сопротивления тела человека сильно продвинулось Р• — перед, РєРѕРіРґР° РІ исследовательский РѕР±РёС…РѕРґ были внедрены измерения сопротивлений РЅР° высоких частотах.

Эти измерения впервые показали, что электрическое сопротивление внутренних органов является преимущественно активным сопротивлением, Р° емкостная составляющая полного сопротивления обусловлена РІ РѕСЃРЅРѕРІРЅРѕРј сопротивлением кожи.  [29]

�змерения электрического сопротивления тела человека обычно проводятся обстоятельно, но противоречия не сглаживаются.

Называется общее значение сопротивления в омах, но не указываются ни площади поверхностей электродов, ни силы, с которыми они давили на кожу.

Если сообщается Рѕ сопротивлении РїСЂРё петле СЂСѓРєР° — СЂСѓРєР°, то РЅРµ уточняется, РІ какой части СЂСѓРєРё — тыльной или ладонной — произошел контакт.  [30]

Страницы:      1    2    3    4

Источник: https://www.ngpedia.ru/id457960p2.html

Электрическое сопротивление тела человека

Конспект по безопасности жизнедеятельности

Значение тока через тело человека сильно влияет на тяжесть электротравм. В свою очередь, сам ток согласно закону Ома определяется сопротивлением тела человека и приложенным к нему напряжением, т.е. напряжением прикосновения.

Сопротивление тела человека является комплексной переменной величиной, имеющей нелинейную зависимость от множества факторов, в том числе от состояния кожи, окружающей среды, центральной нервной системы, физиологических факторов. Электрическое сопротивление различных тканей тела человека не одинаково: кожа, кости, сухожилия и хрящи имеют относительно большое сопротивление, а мышечная ткань, кровь, лимфа, пот и особенно нервные пути, спинной и головной мозг – малое сопротивление.

Электрическое сопротивление тела человека, т.е. сопротивление между двумя электродами, наложенными на поверхность тела, в основном определяется сопротивлением кожи. Кожа состоит из двух основных слоёв: наружного (эпидермис), и внутреннего (дерма). Эпидермис состоит из мёртвых ороговевших клеток, лишён кровеносных сосудов и нервов и поэтому является слоем неживой ткани. Толщина этого слоя 0,05 – 0,2 мм. В сухом и незагрязнённом состоянии его можно рассматривать как диэлектрик, обладающий большим удельным сопротивлением. Дерма состоит из волокон соединительной ткани. В этом слое находятся кровеносные и лимфатические сосуды, нервные окончания, корни волос, потовые и сальные железы. Дерма обладает малым сопротивлением току.

Полное сопротивление тела человека есть сумма сопротивлений тканей, расположенных на пути тока. Основным фактором, определяющим величину полного сопротивления, является состояние кожного покрова в цепи тока. При сухой, чистой и неповреждённой коже сопротивление тела человека, измеренное при напряжении до 15 В, составляет 3…100 кОм. Если на участке кожи, где прикладываются электроды, удалить эпидермис, сопротивление тела составит 500…700 Ом. Если под электродами полностью удалить кожу, то будет измерено сопротивление внутренних тканей, которое составит 300…500 Ом.

Электрическое сопротивление тела человека зависит от ряда факторов. Его могут снизить повреждения рогового слоя, увлажнение кожи, тепловое облучение, повышенная температура воздуха.

Сопротивление наружного слоя кожи уменьшается с увеличением площади электродов и зависит от места их приложения, что объясняется различной толщиной эпидермиса, неравномерным распределением потовых желёз, неодинаковой степенью наполнения кровью сосудов кожи. Повышение напряжения, приложенного к телу человека, вызывает уменьшение его сопротивления, которое при напряжениях более 200 В соответствует сопротивлению внутренних тканей (Rвн).

При оценке опасности поражения электрическим током и расчёте защитных мер в электроустановках сопротивление тела человека (Rh)принимают равным 1 кОм.

Рис. Эквивалентная схема электрического сопротивления тела человека

На рис. 4.1 приведён упрощённый вариант эквивалентной схемы цепи протекания электрического тока через тело человека.

На рисунке обозначено: 1 – электроды; 2 – эпидермис; 3 – внутренние ткани и органы тела человека, включая дерму; Ih – ток, протекающий через тело человека; Uh – напряжение, приложенное к электродам; – активная составляющая сопротивления наружного слоя кожи; – ёмкость условного конденсатора, обклад ка ми которого являются электрод и хорошо проводящие ток ткани тела человека, расположен ные под эпидермисом, а диэлектриком – эпидермис; Rвн – активное сопротивление внутренних тканей, включая дерму.

Из схемы на рис. следует, что комплексное сопротивление тела человека определяется соотношением:

где Xн = 1/ jw Cн – величина ёмкостной составляющей сопротивления тела человека;

w=2p f , f – частота действующего тока.

Для практических применений используют модуль комплексного сопротивления тела человека:

Конспект по безопасности жизнедеятельности

Наибольшее сопротивление электрическому току оказывают

§ 10.2. ЭЛЕКТРИЧЕСКОЕ СОПРОТИВЛЕНИЕ ТЕЛА ЧЕЛОВЕКА

Тело человека, являясь проводником электрического тока, обладает вместе с тем электрическим сопротивлением, однако значение этого сопротивления различно у разных людей. Так, при сухой чистой и неповрежденной коже сопротивление тела человека колеблется от 3000 до 100 000 Ом и более, причем основную часть сопротивления создает кожа человека, тогда как ткани тела обладают весьма малым сопротивлением (300—500 Ом). На сопротивление кожи сильное влияние оказывает ее состояние: наличие царапин, трещин, ссадин. Увлажнение кожи водой или потом, загрязнение кожи (особенно металлической или угольной пылью) сильно снижают сопротивление. На сопротивление кожи сильно влияют площадь контактов и особенно место их прикосновения. Весьма малым сопротивлением обладают участки кожи лица, шеи, рук выше локтя, тыльной стороны кистей рук, подмышечные впадины человека. Сопротивление кожи ладоней и подошв из-за ее загрубелости и мозолистости значительно выше. Сопротивление кожи падает и при длительном прохождении через нее тока, так как ток вызывает нагревание кожи, а это в свою очередь приводит к расширению сосудов, усилению кровообращения и потоотделению. Сопротивление кожи также уменьшается и при повышении напряжения. Например, при напряжениях 50—200 В наступает электрический пробой нечувствительного поверхностного слоя кожи — рогового слоя.

© 2019 Научная библиотека

Копирование информации со страницы разрешается только с указанием ссылки на данный сайт

Тема: какое электрическое сопротивление имеет человеческое тело.

Человеческое тело, как и любое другое тело живого организма, имеет свойство проводить через себя электрический ток. Разные живые ткани в организме имеют различную проводимость (сопротивление). К примеру — кожа, жировая ткань, кости – имеют большое сопротивление, а кровь, мышечная масса и особенно головной и спинной мозг – малое. Кожа имеет большое удельное электрическое сопротивление, что впоследствии и определяет фактическое сопротивление человеческого тела.

Кожа человека, как известно, имеет два слоя:

наружный слой кожи (также ещё называется эпидермис) состоит из несколько слоёв, самый верхний из которых называется роговым и представляет собой множество рядов отмерших и ороговевших клеток. В чистом и сухом виде этот слой можно характеризовать как диэлектрик (он имеет очень большое электрическое сопротивление). Следующий слой эпидермиса (носит название — ростковый) гораздо тоньше рогового и имеет значительно большую электрическую проводимость (меньшее сопротивление).

внутренний слой кожи (называется дерма) представляет собой живую ткань. Данный слой дермы имеет малое электрическое сопротивление.

Электрическое сопротивление обычного человека при условии, что кожа у него чистая, сухая и неповреждённая (измеренное напряжением 15-20 Вольт) лежит в пределах 3 — 100 кОм (1кОм = 1000 Ом), в некоторых случаях и более. Сопротивление тела человека, а именно проводимость между двух электродов, которые касаются поверхности кожи, можно рассматривать как 3 сопротивления включённых последовательно: наружные слои (эпидермиса) представляют собой первое сопротивление, и внутренние слои является вторым и третьим сопротивлением, включающим в себя сопротивления внутреннего слоя кожи и сопротивление внутренних тканей.

Наружное сопротивление человека обладает не только активным сопротивлением, а ещё и ёмкостным, поскольку в самом месте контактирования электродов с человеческим телом образовывается некое подобие конденсатора, в роле обкладок которого являются сами электроды и ткани тела человека, хорошо проводящие электрический ток, что находятся под наружным слоем кожи, ну, а диэлектриком (изолятором между обкладками) в данном случае будет выступать наружный слой кожи (эпидермис).

Ёмкостная составляющая, присутствующая в сопротивлении человека обуславливает влияние, как рода электрического тока, так и его частоты на общую величину сопротивления тела. При частоте 10 — 20 кГц и свыше можно утверждать, что поверхностный слой кожи почти полностью утрачивает своё сопротивление, и общее сопротивление человека в данном случае будет состоять лишь из внутреннего сопротивления тела (сопротивление дермы и внутренних тканей).

Общее состояние кожи в значительной мере оказывает влияние на величину электрического сопротивления человека. При повреждении рогового слоя кожи (царапины, порезы, ссадины и т.д.) происходит снижение сопротивления человека до величины, приближенного к значению внутреннего сопротивления, а это, естественно, повышает опасность поражения электрическим током. Подобное влияние может оказываться и в случае увлажнения кожи водой или потом.

При электрическом переменном токе промышленной частоты (50 герц) берут во внимание только активное сопротивление человека (его тела) и соотносят его с величиной равной 1 кОм. В действительности данное электрическое сопротивление есть величина непостоянная, что имеет нелинейную характеристику и зависит от дополнительных условий, в том числе от параметров электрической цепи, состояния кожи, состояния окружающей среды, физиологии человека и т.д.

Так как сопротивление кожи у одного и того же человека может быть неодинаковое в разных местах и частях тела, то, естественно, на его сопротивление сильно будет влиять конкретное место прикосновения электрических контактов, а также их общая площадь. Величина электрического тока и длительность воздействия на тело оказывают прямое влияние на полное сопротивление человека: с увеличением значения тока и времени его прохождения, сопротивление будет понижаться, потому что происходит местный нагрев участков кожи, а это, само собой, ведёт к расширению сосудов, тем самым усиливая снабжение данного участка тела кровью, увеличения его потоотделение. Увеличение напряжения, воздействующее на тело человека, вызывает понижение сопротивления кожи в 10-ки раз, следовательно, и общее сопротивление человека, снижается до предела 300 — 500 Ом. А это опасно.

Электрическое сопротивление тела человека — это сопротивление току, проходящему по участку тела между двумя электродами, приложенными к поверхности тела человека. Оно складывается из сопротивления наружного слоя кожи и внутреннего сопротивления рук и корпуса.

Рисунок 1 — Электрическая схема человека

— напряжение, приложенное к обоим рукам;

— наружное сопротивление рук;

— емкостное сопротивление рук;

— внутренние сопротивление (из внутреннего сопротивления рук и корпуса).

Тело человека является проводником электрического тока. Разные ткани проводят электрический ток по-разному. Наибольшее сопротивление току оказывает кожа, тогда как ткани тела обладают довольно малым сопротивлением 300-500 Ом.

Кожа состоит из двух основных слоев: наружного — эпидермиса и внутреннего — дермы. Верхний слой кожи роговой (самый толстый) можно рассматривать как диэлектрик. Его удельное сопротивление в 1000 раз превышает сопротивление других слоев кожи и внутренних тканей организма.

У разных людей сопротивление тела различно. При сухой, чистой и неповрежденной коже сопротивление тела человека колеблется от 3000 до 100000 Ом и более.

Очень малым сопротивлением обладает кожа лица, шеи, рук выше локтя, тыльной стороны кистей рук, подмышечные впадины человека.

Расчетное сопротивление человека принято считать 1000 Ом.

Состояние кожи сильно влияет на значение электрического сопротивления тела человека. Так, поврежденный роговой слой имеет наименьшее сопротивление. Порезы, царапины и другие микротравмы могут снизить сопротивление до величины, близкой к величине внутреннего сопротивления, поэтому увеличивается опасность поражения человека электрическим током. Такое же влияние оказывает увлажнение кожи водой или потом, а также загрязнение токоведущей пылью и грязью.

Основные факторы, влияющие на исход поражения электрическим током

1 величина тока

2. величина напряжения

3. род и частота тока

4. путь тока (путь замыкания)

5. время действия (длительность воздействия)

6. сопротивление человека (индивидуальные свойства человека, физиологическое и психологическое состояние).

7. окружающая среда;

Путь тока и длительность воздействия электрического тока являются самыми основными факторами, влияющими на исход поражения электрическим током.

Влияние величины тока на исход поражения электрическим током

Сила или величина электрического тока, проходящего через тело человека, воздействует следующим образом:

Ощутимый ток (0,6-1 1,5 мА) вызывает слабый зуд и легкое покалывание, он не опасен для жизни, но при длительном воздействии отрицательно скрывается на здоровье человека.

Неотпускающий ток (10- 15мА) вызывает сильною боль, судороги усиливаются, пострадавший не может разжать руку, в которой находится токоведущая часть.

Ток в 25-50 мА действует не только на мышцы рук, но и на мышцы туловища, длительное воздействие такого тока может привести к прекращению дыхания и даже к смерти.

Фибрилляционный ток (100 мА и более). Такой ток очень опасен: через 1-2 секунды после начала его действия, начинаются частые сокращения волокон сердечной мышцы (фибрилляция), прекращается движение крови в сосудах и наступает смерть. Если действие тока кратковременно и не вызывает повреждение сердца, то после отключения тока сердце самостоятельно возобновляет нормальную деятельность, а для восстановления дыхания требуется немедленная помощь (искусственное дыхание).

Влияние рода тока и частоты на исход поражения электрическим током При невысоких напряжениях (до 100 В) постоянный ток примерно в 3-4 раза менее опасен, чем переменный частотой до 50 Гц. При напряжениях 400-500В опасность их сравнивается, а при более высоких напряжениях постоянный ток даже опаснее переменного. С увеличением частоты тока до 50Гц опасность поражения несколько увеличивается, а при частоте свыше 50 Гц опасность поражения уменьшается. Токи высокой частоты 450-500 Гц сохраняют опасность ожогов.

При повышении частоты переменного тока (начиная с 1000-2000 Гц) опасность электрического тока снижается. При частоте 450-500 Гц опасность исчезает (это объясняется поверхностным эффектом — ток высокой частоты проходит по нечувствительной поверхности кожи).

Постоянный ток в 4-5 раз безопаснее переменного при напряжениях до 250-300 В.

При более высоких напряжениях постоянный ток опаснее переменного.

Влияние пути тока в теле человека на исход поражения электрическим током

При прикосновении человека к токоведущим частям путь тока может быть различным. Всего существует 18 вариантов путей замыкания тока через человека.

Если на пути тока оказываются жизненно важные органы: сердце, легкие, головной мозг, то опасность тяжелого исхода весьма велика. Если ток проходит иными путями, то воздействие его на жизненно важные органы может быть рефлекторным, т.е. через центральную нервную систему. Благодаря этому вероятность тяжелого исхода резко уменьшается, поскольку путь тока зависит от того, каким участком тела пострадавший прикасается к токоведущим частям. Его влияние на исход поражения проявляется еще и потому, что сопротивление на разных участках тела различное.

В теле человека наиболее частые пути тока: рука-рука, правая рука — ноги, левая рука — ноги, нога-нога, голова — ноги.

Наиболее опасен продольный путь тока через тело человека: рука-нога, голова-нога.

Менее опасен поперечный — рука-рука, нога-нога.

Влияние длительности прохождения тока через тело человека на исход поражения

Чем продолжительнее действие тока на организм, тем выше вероятность тяжелого или смертельного исхода. Такая зависимость объясняется тем, что с увеличением времени воздействия тока на живую ткань возрастает значение этого тока (за счет уменьшения сопротивления тела). Накапливаются отрицательные последствия воздействия тока на организм, и резко возрастает совпадения момента прохождения тока через сердце с уязвимой для него фазой сердечного цикла (фибрилляция сердца).

Влияние внешней среды на механизм поражения электрическим током

Присутствие в воздухе в помещениях ряда производств химически активных, токсичных газов, попадающих в организм человека, снижает электрическое сопротивление тела человека.

Во влажных и сырых помещениях происходит увлажнение кожи. Влага, попадая на кожу, растворяет находящиеся на ней минеральные вещества, жирные: кислоты, делая кожу электропроводящей.

При работе в помещениях с высокой температурой окружающей среды на теле человека происходит усиленное потоотделение. Пот хороший проводник электрического тока. Работа в таких условиях повышает воздействие электрического тока на организм человека. В ряде случаев имеет место загрязнения кожи различными веществами, хорошо проводящими электрический ток.

В отдельных производственных помещениях возникает шум и вибрации, отрицательно действующие на организм человека: повышается кровяное давление, нарушается режим дыхания. Эти факторы, а также недостаток освещения вызывают замедление психической реакции, понижает внимание и приводит к авариям, несчастным случаям и электротравмам.

Влияние индивидуальных свойств человека на исход поражения электрическим током

Установлено, что здоровые и физически крепкие люди переносят электрические удар легче, чем больные и слабые.

Повышенной восприимчивостью к электрическому току обладают лица, страдающие болезнями кожи, сердечно-сосудистой системы, органов внутренней секреции, легких, неврозами.

Поэтому правилами техники безопасности при эксплуатации электроустановок предусматривается отбор по состоянию здоровья персонала.

Исследовательская работа, «Сопротивление тела человека»

Муниципальное бюджетное общеобразовательное учреждение

«Бардымская средняя общеобразовательная школа №2»

Муниципальная научно-практическая конференция учащихся

«Юный исследователь»

ЭЛЕКТРИЧЕСКОЕ СОПРОТИВЛЕНИЕ

ТЕЛА ЧЕЛОВЕКА

Работу выполнил:

Мансуров Данил

Ученик 9б класса МБОУ «Бардымская СОШ №2»

Руководитель:

Халитова Гюзель Феликсовна

Учитель физики МБОУ «Бардымская СОШ №2»

с.Барда

2016 г.

ОГЛАВЛЕНИЕ

Используя электротехнические изделия в быту или на производстве, человек может попасть под действие электрического тока. Одним из факторов, влияющие на исход поражения человека электрическим током, является сопротивление тела человека. Поэтому проблема защиты человека от воздействия электрического тока была и остаётся актуальной.

Все чаще учащихся проявляют интерес к изучению физики тела человека. Все чаще они пробуют объяснить те или иные изменения состояния человека с точки зрения науки. Но в курсе физики, изучаемом в школе, практически не уделяется внимания физическим параметрам, характеризующим человека. Законы рассматриваются в основном на неживых объектах. Именно поэтому было принято решение провести исследование одного из физических параметров, а именно, сопротивление тела человека. И на этих исследованиях показать, что, причинно-следственная связь явлений имеет всеобщий характер и что, все явления, происходящие в окружающем нас мире, взаимосвязаны.

Существуют теоретические утверждения о зависимости сопротивления тела от кожного покрова, не исключено и влияние пути протекания электрического тока по телу, а также влияние физического и психоэмоционального состояния [2].

Цель исследования: провести измерение сопротивления тела человека электрическому току и выяснить от каких факторов оно зависит.

Объект исследования: учащиеся МБОУ БСОШ №2 (в возрастной категории от 6 до 9 класса) , семья Мансуровых.

Предмет исследования – руки человека.

Для достижения цели поставлены следующие задачи исследования:

  • научиться работать с научной литературой, отбирать, анализировать, систематизировать информацию;

  • научиться правильно пользоваться мультиметром;

  • исследовать зависимость сопротивления тела человека:

  1. от состояния кожного покрова,

  2. от времени суток,

  3. от физиологических факторов .

В процессе данной исследовательской работы использовались следующие теоретические и эмпирические методы:

  • сравнение и сопоставление теоретических данных с результатами эксперимента;

  • анализ различных информационных источников и результатов исследования по данному вопросу;

  • систематизация приведения в систему полученных теоретических и практических знаний.

  • наблюдение за экспериментом

  • исследование – выявление зависимости сопротивления тела от различных факторов.

Новизна исследований заключалась в проведении измерение сопротивления тела человека:

  • в разное время суток,

  • с различным состоянием поверхности кожи,

  • с различным эмоциональным состоянием,

  • у детей и подростков разного возраста (от12 до 16 лет)

  • у детей и подростков разных полов.

В ходе работы, рассматривались вопросы, направленные на развитие интереса к физике, к экспериментальной деятельности, формирование умений работать со справочной литературой. Я познакомился с методами исследования в физике и биологии, получил краткие данные о медицинской и биологической аппаратуре. Объяснение отдельных процессов, происходящих в живых организмах, на основе физических законов помогли мне установить причинно-следственные связи, существующие в живой и неживой природе, сформировали интерес не только к физике, но и биологии. Навыки, полученные при работе с измерительным прибором, при постановке эксперимента пригодятся в дальнейшем в научно-технической деятельности.

1. 1. Электричество и кожа

Существование и развитие человека невозможно без непрерывного взаимодействия с окружающей средой. Влияние внешней среды на человека обычно рассматривается на примере действия электрического тока и магнитного поля. Причем это не случайно. Энергия любого из этих факторов, так или иначе, преобразуется в электрическую, которая, взаимодействуя с электричеством человека, обуславливает реакцию человека на действие внешнего фактора[6].

Преобразование энергии взаимодействующих факторов в электрическую подчиняется определенной передаточной функции. Основные процессы преобразования, описываемые передаточной функцией, происходят через кожу. Кожа является источником информации о состоянии органов и тканей человека и в то же время – защитной оболочкой человека от вредного воздействия среды.

Кожа, осуществляющая связь в системе «среда – человек» (рис. 1.1), представляет собой трехкомпонентную структуру, образованную эпидермисом, дермой и подкожной жировой клетчаткой, которые находятся в функциональном разрезе. Самым тонким слоем является эпидермис. Несмотря на незначительные размеры, он обладает наиболее ответственными функциями – защитной и информационной. Информация необходима для саморегуляции ряда биофизических процессов в организме, прежде всего тепловых и биоэлектрохимических.

Рис.1.1. Кожа человека

Эпидермис плоский, тонкий, ороговевший слой, представляет собой пограничную часть с разнообразными сложными барьерно-информативными функциями. Одна из основных функций – защита от проникновения в организм чужеродных, не свойственных ему микробов, аэрозольной пыли. Он способствует защите тканей и органов от проникновения ультрафиолетового и коротковолнового рентгеновского излучения. Структурные особенности эпидермиса обеспечивают ему высокую упругость, эластичность. Он имеет большую механическую прочность, что позволяет ему выдерживать большие механические нагрузки. Обладая высокими регенерационными свойствами, способен при повреждениях быстро восстанавливаться. Благодаря удивительным и многообразным видам электропроводимости он имеет исключительно высокую рецепторную защитную способность[2].

Кожу многие ученые представляют, как топографическую связь отдельных участков эпидермиса со всеми органами человека. В эпидермисе находятся акупунктуры – точки и участки кожи, обладающие отличным от основного состава эпидермиса значением проводимости. Значит, есть различие и в свойствах этих точек. Через эти зоны в основном и осуществляется связь эпидермиса с внутренними органами. Возникновение электрической цепи через область эпидермиса в акупунктурных зонах может привести к смертельному исходу даже при очень маленьком напряжении. В то же время очень популярно воздействие на эти точки иглами с целью лечения или усиления некоторых функций организма – иглотерапия.

Свойства кожи уникальны и удивительны. Уже давно было обнаружено, что клетки чистой кожи убивают болезнетворные бактерии и микробы, попадающие на ее поверхность, на воздухе, и в то же время через мокрую кожу могут свободно проходить эти же микробы. Чем это вызвано?

Эпидермис – поверхностный слой кожи относится к диэлектрикам, обладающим огромным удельным сопротивлением и большим значением диэлектрической проницаемости. Под влиянием разности температур внутренних органов и окружающей среды возникает диффузия «электрического газа». При прохождении газа через место ранения, обладающего высоким удельным сопротивлением и большой диэлектрической проницаемостью, появляется статическое электричество. Напряженность поля может достигнуть десятка киловольт на 1 квадратный сантиметр. При такой напряженности клеточные мембраны разрушаются и бактерии погибают. Для разрушения нейрона или клетки достаточна электрическая энергия поля в пределах 10-20 Дж. Это свидетельствует о том, что кожа является своеобразным электростатическим фильтром, подобным электростатическому фильтру, применяемому в системах жизнеобеспечения для замкнутых помещений, представляя собой стерилизатор. Но все это происходит при условии, что сопротивление кожи поддерживается на очень высоком уровне. При наличии воды на коже или повышенной влажности кожи такое электростатическое поле возникнуть не может – нет и «стерилизатора». Следовательно, электричество человека служит очень хорошим стражем от поражения микроорганизмами – бактериями окружающей человека воздушной среды[5].

1.2. Электрические свойства тела человека

Человек – существо многогранное: он покорил высочайшие горные вершины, опустился в самые глубокие точки Мирового океана, побывал на Луне, расщепил атомное ядро. Но чаще всего мы не задумываемся, а что же мы представляем собой, что мы можем сделать, какими возможностями и ресурсами обладаем? Человеческий организм представляет собой сложные биологические системы, поэтому, изучая происходящие в нем процессы, мы обратились к методам, которые используются в таких точных науках, как физика и химия. Ведь человек состоит из атомов и молекул, подчиняющихся физическим и химическим закономерностям. По нервному волокну распространяется электрический импульс. Профессор кафедры анатомии в Болонье (Италия) Луиджи Гальвани в книге «Трактат о силах электричества при мышечном движении» пишет: «Я разрезал и препарировал лягушку… Когда один из моих помощников острием скальпеля случайно очень легко коснулся внутренних бедренных нервов этой лягушки, то немедленно все мышцы конечностей начали так сокращаться, что казались впавшими в сильнейшие тонические судороги… Я зажегся страстным желанием исследовать это явление…» Но ответ на этот вопрос дал соотечественник Гальвани – Алессандро Вольта: живой организм проводит, пропускает через себя электрический ток. Именно Вольта и создал первый источник. Такой источник тока можно продемонстрировать. Нужно взять по пять пластинок из меди и цинка размером 30х30х4 мм и сложить их стопкой, чередуя и перекладывая промокательной бумагой, смоченной крепким раствором поваренной соли. Если взять столбик мокрыми пальцами за торцы, то почувствуешь слабый, но явственный электрический удар! Если этот элемент подключить к гальванометру, он покажет наличие тока в цепи. Прошел 121 год после статьи Гальвани, и в 1912 г. было обнаружено, что внутри человеческого организма протекают токи, хотя и очень слабые. Исследователи доказали, что любой процесс внутри человека: работа сердца и мозга, прохождение нервных сигналов, мышечные сокращения – сопровождаются биологическими электрическими сигналами.

Электропроводность – один из параметров, характеризующих жизненную деятельность живого существа. Известно, что с возникновением живого организма любого вида начинаются биоэлектрические явления, которые прекращаются при гибели живого существа. Человек при этом не является исключением. Тело человека представляет собой по своим электрофизическим свойствам соленый раствор (раствор электролита)[2].

1.3. Сопротивление тела человека

Проводимость живой ткани в отличие от обычных проводников обусловлена не только ее физическими свойствами, но и сложнейшими биохимическими и биофизическими процессами, присущими лишь живой материи. В живой ткани нет свободных электронов, и поэтому она не может быть уподоблена металлическому проводнику, электрический ток в котором представляет собой упорядоченное движение свободных электронов.

Большинство тканей тела человека содержит значительное количество воды (до 65% по весу). Поэтому живую ткань можно рассматривать как электролит, обладающий ионной проводимостью. Иначе говоря, можно полагать, что перенос электрических зарядов в живой ткани осуществляется не свободными электронами, как это имеет место в металлических проводниках, а заряженными атомами или группами атомов – ионами.

В живой ткани наблюдается явление межклеточной миграции (перемещение) энергии, т.е. резонансный перенос энергии электронного возбуждения между возбужденной и невозбужденной клетками. Поэтому можно предположить, что живая ткань обладает также электронно-дырочной проводимостью, свойственной полупроводникам, в которых перенос зарядов осуществляется электронами проводимости и дырками.

Таким образом, тело человека можно рассматривать как проводник особого рода, имеющий переменное сопротивление и обладающий в какой-то мере свойствами проводников первого рода (полупроводники) и второго рода (электролиты).

Из этих данных следует, что кожа обладает очень большим удельным сопротивлением, которое является главным фактором, определяющим сопротивление тела человека в целом.

Следовательно, сопротивление тела человека является переменной величиной, имеющей нелинейную зависимость от множества факторов, в том числе от состояния кожи, параметров электрической цепи, физиологических факторов и состояния окружающей среды.

Сопротивление тела человека можно условно считать состоящим из трех последовательно включенных сопротивлений (рис 1.3.): двух одинаковых сопротивлений наружного слоя кожи, т. е. эпидермиса, 2Rн (которые в совокупности составляют так называемое наружное сопротивление тела человека) и одного, называемого внутренним сопротивлением тела Rв (которое включает в себя сопротивление внутренних слоев кожи и сопротивление внутренних тканей тела).

Рис. 1.3. Эквивалентная схема замещения сопротивления тела человека

Сопротивление наружного слоя кожи Rн состоит из активного и емкостного сопротивлений, включенных параллельно. Полное сопротивление наружного слоя кожи Rн зависит от площади электродов, частоты тока, а также от значения приложенного напряжения и при площади электродов в несколько квадратных сантиметров может достигать весьма больших значений.

Внутреннее сопротивление тела считается чисто активным, хотя, оно также обладает емкостной составляющей. Внутреннее сопротивление Rв практически не зависит от площади электродов, частоты тока, а также от значения приложенного напряжения и равно примерно 500 – 700 Ом [5].

Омметр – это измерительный прибор специализированного назначения, предназначенный для определения сопротивления электрического тока. Так как сопротивление выражается в Омах (Ом), то и прибор, его измеряющий получил название омметра.

Омметр – это прибор непосредственного отсчета. Его основная функция – определение активных сопротивлений электрического тока. Как правило, омметр преобразует переменный ток в постоянный и производит измерения. Однако некоторые модели могут измерять сопротивление непосредственно переменного тока, без его преобразования. Омметры являются довольно полезными, а в ряде случаем и незаменимыми приборами, в зависимости от предъявляемых требований к диапазону производимых ими измерений.

Цифровые омметры. Микроомметр MOM600A представляет собой измерительный мост с автоматическим уравновешиванием. Уравновешивание производится цифровым управляющим устройством методом подбора прецизионных резисторов в плечах моста, после чего измерительная информация с управляющего устройства подаётся на блок.

Аналоговые электронные омметры. Принцип действия электронных омметров основан на преобразовании измеряемого сопротивления в пропорциональное ему напряжение с помощью операционного усилителя. Измеряемый объект включается в цепь обратной связи (линейная шкала) или на вход усилителя.

Действие магнитоэлектрического омметра основано на измерении силы тока, протекающего через измеряемое сопротивление при постоянном напряжении источника питания.

Мультиме́тр (от англ. multimeter), те́стер (от англ. test — испытание), аво́метр (от ампервольтомметр) — комбинированный электроизмерительный прибор, объединяющий в себе несколько функций. В минимальном наборе включает функции вольтметра, амперметра и омметра. Иногда выполняется мультиметр в виде токоизмерительных клещей. Существуют цифровые и аналоговые мультиметры.

Мультиметр может быть как лёгким переносным устройством, используемым для базовых измерений и поиска неисправностей, так и сложным стационарным прибором со множеством возможностей.

3.1. Исследование зависимости сопротивления тела человека от состояния кожного покрова

При исследовании зависимости сопротивления кожного покрова были использованы следующие приборы и материалы: мультиметр М-830, (см. приложение рис. 1) дистиллированная и подсоленная вода, салфетки.

Порядок выполнения работы:

1. С помощью мультиметра измерили сопротивление своего тела при чистой, сухой и неповрежденной коже. Электроды приложили от ладони одной руки к ладони другой руки Rлп. Повторили измерения 10 раза и вычислили среднее значение сопротивления. Полученные результаты занесли в таблицу 1(см. приложение на стр.18).

2. Дистиллированной водой увлажнили руки, провели измерения, сопротивление уменьшилось. Данные измерений занесли в таблицу 2(см. приложение на стр.18).

3. Сильно подсоленной водой увлажнили руки, провели измерения, сопротивление тела понизилось еще больше. Данные измерений занесли в таблицу 3(см. приложение на стр.18).

Сопротивление кожи понижается при её увлажнении даже в том случае, если влага обладает большим удельным сопротивлением. Влага, попавшая на кожу, растворяет находящиеся на ее поверхности минеральные вещества и жирные кислоты, выведенные из организма вместе с потом и кожным салом, и становится более электропроводной. Пот хорошо проводит электрический ток, поскольку в его состав входят вода и растворенные в ней минеральные соли, а также некоторые продукты обмена веществ. Он выделяется на поверхность кожи по выводным протокам – тонким трубочкам, пронизывающим всю толщу кожи.

Выделение пота происходит непрерывно, даже на холоде, но особенно обильно при высокой температуре окружающего воздуха, напряжённой физической работе, местном нагреве кожи, болезни человека и т. п.[3].

Следовательно, работа в условиях, вызывающих интенсивное потовыделение, увеличивает опасность поражения человека током, работа сырыми руками или в условиях, вызывающих увлажнение каких-либо участков кожи, создает предпосылки для тяжелого исхода в случае попадания человека под напряжение.

4. Соскоблили роговой слой на сухих участках кожи, где прикладываются электроды, измерили сопротивление, оно снизилось еще больше.

Полученные результаты занесли в таблицу 4(см. приложение на стр.19).

Порезы, царапины, ссадины и другие микротравмы, т.е. повреждения рогового слоя – снижают сопротивление тела человека что, бесспорно, увеличивает опасность поражения током человека.

5. Потерли руки пылью, взятой от мебели. Приложили электроды на загрязненную кожу рук и измерили сопротивление. Полученные результаты занесли в таблицу 5(см. приложение на стр.19).

По результатам исследования, используя Microsoft PowerPoint построили диаграмму «Исследование зависимости сопротивления тела человека от состояния кожного покрова» (см. приложение рис.2)

Загрязнение кожи различными веществами, в особенности хорошо проводящими ток (металлическая или угольная пыль, окалина и т. п.), сопровождается снижением ее сопротивления, подобно тому, как это наблюдается при поверхностном увлажнении кожи. Кроме того, токопроводящие вещества, проникая в выводные протоки потовых и сальных желез, создают в коже длительно существующие токопроводящие каналы, резко понижающие ее сопротивление.

Таким образом, токарь по металлу, шахтер и лица других специальностей, у которых руки загрязняются токопроводящими веществами, подвержены большей опасности поражения током, чем лица, работающие сухими чистыми руками.

Вывод: сопротивление кожи, а, следовательно, и тела в целом резко уменьшается при наличии влаги на ее поверхности, сильном потовыделении, загрязнении и повреждении ее рогового слоя.

3.2. Исследование зависимости сопротивления тела человека от времени суток

Сопротивление тела человека у разных людей различно. Различным оказывается оно и у одного и того же человека в разное время суток. В качестве испы-

туемого выступил Данил Мансуров. Измерения проводились: рано утром – после пробуждения R1, после завтрака R2, в обед R3, вечером R4, ночью R5.

Данные измерений занесли в таблицу 6(см. приложение стр.19).

По полученным данным построили диаграмму (см. приложение рис.3).

Вывод: в результате этого исследования выяснено, что наименьшим сопротивлением тело обладает ночью и рано утром, наибольшим – сразу после завтрака (зарядки) и ближе к вечеру. Это объясняется тем, что утором организм отдохнул и после завтрака получил большой запас энергии. Следовательно, завтрак – полезный и важный прием пищи, пренебрегать которым не стоит. Завтрак является своеобразным аккумулятором, который заряжает человека энергией на весь день.

3.3. Исследование зависимости сопротивления тела человека от физиологических факторов

На значение сопротивления влияют и другие факторы, хотя и в значительно меньшей степени, например пол и возраст.

1. Для проведения опыта были приглашены близкие мне люди разного пола и возраста – мужчина (папа), женщина (мама), девочка (сестра – 9 лет) и я (мальчик 15 лет) . Измерения проводились с помощью мультиметра при одинаковых начальных условиях – ежедневно в 18.00 по местному времени в течении 3 дней. Электроды прикладывались от одной руки к другой. Полученные результаты занесли в таблицу 7(см. приложение стр.20) и построили диаграмму (см. приложение рис.4).

У мужчин сопротивление тела больше, чем у женщин, а у девочек – меньше, чем у мальчиков. У взрослых больше, чем у детей. Объясняется это, очевидно, тем, что у одних людей кожа тоньше и нежнее, у других – толще и грубее. Расчитали среднее значение сопротивления и отобразили данные в виде диаграммы (см. приложение рис. 4)

2. Для доказательства того, что сопротивление человека меняется с возрастом и полом провели эксперименты с учащимися разных классов (от 6 по 9). Были приглашены девочки и мальчики, юноши и девушки в равных количествах с разной возрастной категории 5 девушек (девочек) и 5 юношей (мальчиков). Всего испытуемых – 60 человек. Результаты измерений занесли в таблицу 8(см. приложение стр.21).

Как видно из опытов кожа учащихся старших классов обладают большим сопротивление нежели младших, это объясняется тем, что с возрастом кожа становится толще, её защитные функции становятся более совершенными. По данным исследований построили диаграмму (см. приложение рис.5)

При проведении было замечено что сопротивление кожи девочек меньше нежели сопротивление мальчиков, что обусловлено физиологическими особенностями – кожа мальчиков имеет более толстый роговой слой, а дерма богаче коллагеном, поэтому она более плотная и упругая (см. приложение рис. 5). Хотя были случаи, когда сопротивление кожи мальчиков было очень маленьким.

3. Для определения зависимости сопротивления человеческого тела от настроения, проверялся мальчик Дамир — 10 мес. Измерили сопротивление тела прикладывая электроды к ладоням испытуемого. Результаты внесли в таблицу 9(см. приложение стр.22).

По полученым результатам построили диаграмму (см. приложение рис.6).

Максимальное сопротивление тела оказалось у Дамира, когда он был зол и раздражен. Минимальным соротивлением обладал Дамир , когда он был грустным. У Дамира , когда он был в радостном настроениии сопротивление изменилось мало, но повысилось. Испытуемый Дамир подвергался и физическим раздражениям, возникающим неожиданно это световые, звуковые, болевые.Все перечисленные раздражения привели к снижению сопротивления на 20-50%

Вывод: на сопротивление кожи человека влияют физиологические особенности организма, а также эмоциональное состояние. С возрастом наша кожа становится более защищенной.

При анализе полученного материала выявляется четко выраженная зависимость сопротивления тела человека от:

— состояния кожного покрова,

— физического и психического состояния человека,

— от времени суток.

Таким образом, наша гипотеза о зависимости сопротивления тела от кожного покрова, физиологических факторов была подтверждена экспериментальным путем.

На величину сопротивления, а, следовательно, и на исход поражения электрическим током, большое влияние также оказывает повышенная потливость кожного покрова, переутомление, нервное возбуждение, физические раздражители.

Все перечисленные факторы приводят к резкому уменьшению сопротивления тела человека (до 800–1000 Ом). Поэтому, даже сравнительно небольшие напряжения могут привести к поражению электрическим током. Также значение полного сопротивления тела человека зависит и от параметров электрической цепи – места приложения электродов к телу человека, значений тока и приложенного напряжения, рода и частоты тока, площади электродов, длительности прохождения тока. Это дальнейшее продолжение изучение данной темы.

Каждый работающий с электро-радиоаппаратурой должен помнить, что человеческий организм поражает не напряжение, а величина тока. При неблагоприятных условиях даже низкие напряжения (30–40 В) могут быть опасными для жизни. Если сопротивление тела человека равно 700 Ом, то опасным будет напряжение 35 В.

Данная работа может быть применена для профилактики несчастных случаев при поражении электрическим током, а также поможет при подготовке к проведению занятий по ОБЖ.

  1. Беркинблит М.Б. и др. Электричество в живых организмах. – М.: Наука, 1988.

  2. Богданов К.Ю. Физика в гостях у биолога. – М., 1986.

  3. Долин П.А. Основы техники безопасности в электроустановках. – М.: «Знак», 2000.

  4. Охрана труда в энергетике / под ред. Б.А. Князевского. – М.: “Энергоатомиздат”, 1985. – 200с.

  5. Энциклопедия для детей [том 16]. Физика Ч.1 Биография физиков. Путешествия вглубь материала. Механическая картина мира. – 2-е изд. Испр. /ред. Коллегия: М. Аксёнова, В. Володин, А. Элиович и др. – М.: Мир энциклопедий Аванта+, 2007. – 448с.: ил.

  6. Энциклопедия для детей [том 16]. Физика Ч.2 Электричество и магнетизм. Термодинамика и квантовая механика. Физика ядра и элементарных частиц. /ред. Коллегия: М. Аксёнова, А. Элиович, В. Санюк и др. – М.: Мир энциклопедий Аванта+, 2007. – 432с.: ил

  7. Информационный сайт

http://ftemk.mpei.ac.ru/bgd/bgd_.htm#deistvie_toka – Безопасность жизнедеятельности Московский энергетический институт (ТУ) Кафедра инженерной экологии и охраны труда.

Рис.1.1 Мультиметр М830BZ

Таблица 1

опыта

1

2

3

4

5

6

7

8

9

10

Rлп, кОм

1,9

2

1,8

1,9

1,8

1,8

1,9

1,9

1,8

1,9

Rср, кОм

1,87

Таблица 2

Таблица 3

опыта

1

2

3

4

5

6

7

8

9

10

Rлп, кОм

1,3

1,2

1,1

1,4

1,2

1,2

1,1

1,1

1,3

1,1

Rср, кОм

1,2

Таблица 4

Таблица 5

Рис.2. Зависимости сопротивления тела человека от состояния кожного покрова

. Таблица 6

R1,

кОм

Rср1, кОм

R2 , кОм

Rср2, кОм

R3,

кОм

Rср3,

кОм

R4,

кОм

Rср4,

кОм

R5,

кОм

Rср5,

кОм

1

1,6

1,6

1,7

1,7

1,6

1,63

1,7

1,7

1,5

1,51

2

1,7

1,7

1,6

1,6

1,5

3

1,8

1,6

1,5

1,8

1,5

4

1,6

1,8

1,65

1,8

1,4

5

1,5

1,9

1,7

1,7

1,6

6

1,5

1,6

1,7

1,8

1,6

7

1,5

1,6

1,65

1,6

1,5

Рис.3. Зависимости сопротивления тела человека от времени суток.

Таблица 7

мужчина

женщина

мальчик

девочка

№ опыта

R1,

кОм

Rср1, кОм

R2 , кОм

Rср2, кОм

R3,

кОм

Rср3,

кОм

R4,

кОм

Rср4,

кОм

1

1,9

1,97

1,8

1,72

1,9

1,84

1,5

1,48

2

2

1,7

1,8

1,5

3

1,9

1,8

1,8

1,4

4

2

1,6

1,9

1,5

5

1,9

1,7

1,8

1,5

6

1,9

1,6

1,9

1,5

7

2

1,7

1,8

1,5

Рис.4. Зависимости сопротивления тела человека от физиологических факторов

Таблица 8

5 класс

6 класс

7 класс

8 класс

9 класс

10 класс

№ оп.

R5,

кОм

Rср5,

кОм

R6,

кОм

Rср6,

кОм

R7,

кОм

Rср7,

кОм

R8,

кОм

Rср8,

кОм

R9,

кОм

Rср9,

кОм

R10,

кОм

Rср10,

кОм

1

1,2

1,23

1,1

1,12

1,1

1,21

1,3

1,44

1,4

1,46

1,4

1,51

2

1,2

1,2

1,2

1,3

1,3

1,5

3

1,3

1

1

1,6

1,6

1,6

4

1,2

1,2

1,2

1,4

1,4

1,5

5

1,3

1

1,2

1,5

1,5

1,5

6

1,3

0,8

1,3

1,6

1,6

1,6

7

0,9

1,3

1,4

1,3

1,2

1,4

8

1,3

1,1

1,1

1,5

1,6

1,6

9

1,2

1,2

1,3

1,4

1,6

1,6

10

1,4

1,3

1,3

1,5

1,4

1,4

Рис.5. Зависимость сопротивления человеческого тела от возраста

Таблица 9

раздражение

радость

грусть

злость

№ опыта

R1,

кОм

Rср1, кОм

R2 , кОм

Rср2, кОм

R3,

кОм

Rср3,

кОм

R4,

кОм

Rср4,

кОм

1

0,9

0,88

0,9

0,78

0,7

0,67

0,8

0,83

2

1

0,7

0,6

1

3

0,8

0,7

0,7

0,9

4

0,8

0,9

0,6

0,8

5

0,9

0,8

0,6

0,8

6

0,8

0,8

0,6

0,7

7

0,9

0,8

0,7

0,7

8

1

0,6

0,7

0,8

9

0,9

0,7

0,7

0,9

10

0,8

0,9

0,8

0,9

Рис.6. Зависимость сопротивления человеческого тела от настроения

Рис.8. Зависимость сопротивления человеческого тела от окружающей среды

Рис. 7. Факторы влияющие на исход поражения электрическим током

3.4: Закон Ома (снова) — Workforce LibreTexts

Принцип «текущее убивает» по существу верен. Это электрический ток, который сжигает ткани, замораживает мышцы и вызывает фибрилляцию сердца. Однако электрический ток не возникает сам по себе: должно быть доступное напряжение, чтобы заставить электроны проходить через жертву. Тело человека также оказывает сопротивление току, что необходимо учитывать.

Принимая закон Ома для напряжения, тока и сопротивления и выражая его через ток для заданных напряжения и сопротивления, мы получаем следующее уравнение:

Величина тока, протекающего через тело, равна величине напряжения, приложенного между двумя точками этого тела, деленному на электрическое сопротивление, оказываемое телом между этими двумя точками.Очевидно, что чем больше напряжения доступно, чтобы заставить электроны течь, тем легче они будут проходить через любое заданное сопротивление. Следовательно, опасность высокого напряжения: высокое напряжение означает возможность протекания через ваше тело большого количества тока, который может вас травмировать или убить. И наоборот, чем большее сопротивление тело оказывает току, тем медленнее электроны будут течь при любом заданном напряжении. Насколько опасно напряжение, зависит от того, какое полное сопротивление в цепи препятствует потоку электронов.

Сопротивление тела не является фиксированной величиной. Это варьируется от человека к человеку и время от времени. Существует даже метод измерения содержания жира в организме, основанный на измерении электрического сопротивления между пальцами рук и ног. Различное процентное содержание жира в организме дает разное сопротивление: всего одна переменная, влияющая на электрическое сопротивление в теле человека. Чтобы методика работала точно, человек должен регулировать потребление жидкости за несколько часов до теста, что указывает на то, что гидратация тела является еще одним фактором, влияющим на электрическое сопротивление тела.

Сопротивление тела также зависит от того, как происходит контакт с кожей: от руки к руке, от руки к ноге, от ступни к ступне, от руки к локтю и т. Д.? Пот, богатый солями и минералами, является отличным проводником электричества, будучи жидкостью. То же самое и с кровью с таким же высоким содержанием проводящих химикатов. Таким образом, контакт с проводом потной рукой или открытой раной будет оказывать гораздо меньшее сопротивление току, чем контакт с чистой сухой кожей.

Измеряя электрическое сопротивление чувствительным измерителем, я измеряю сопротивление приблизительно 1 миллион Ом (1 МОм) двумя руками, держась за металлические щупы измерителя между пальцами.Измеритель показывает меньшее сопротивление, когда я сильно сжимаю щупы, и большее сопротивление, когда я держу их слабо. Я сижу за компьютером и печатаю эти слова, мои руки чистые и сухие. Если бы я работал в жаркой, грязной промышленной среде, сопротивление между моими руками, вероятно, было бы намного меньше, представляя меньшее сопротивление смертоносному току и большую опасность поражения электрическим током.

А какой ток вреден? Ответ на этот вопрос также зависит от нескольких факторов.Химический состав тела человека оказывает значительное влияние на то, как электрический ток влияет на человека. Некоторые люди очень чувствительны к току, испытывая непроизвольное сокращение мышц из-за разряда статического электричества. Другие могут получить большие искры от разряда статического электричества и почти не почувствовать его, не говоря уже о мышечном спазме. Несмотря на эти различия, с помощью тестов были разработаны приблизительные руководящие принципы, которые показывают, что для проявления вредных эффектов требуется очень небольшой ток (опять же, информацию об источнике этих данных см. В конце главы).Все текущие значения даны в миллиамперах (миллиампер равен 1/1000 ампер):

Таблица воздействия электричества на тело

«Гц» означает единицу Гц, , меру того, насколько быстро меняется переменный ток, величина, также известная как частота . Таким образом, столбец цифр, обозначенный «60 Гц переменного тока», относится к току, который меняется с частотой 60 циклов (1 цикл = период времени, когда электроны текут в одном направлении, а затем в другом) в секунду.Последний столбец, обозначенный «10 кГц переменного тока», относится к переменному току, который совершает десять тысяч (10 000) возвратно-поступательных циклов каждую секунду.

Имейте в виду, что эти цифры являются приблизительными, поскольку люди с разным химическим составом тела могут реагировать по-разному. Было высказано предположение, что ток через грудную клетку всего 17 мА переменного тока достаточно, чтобы вызвать фибрилляцию у человека при определенных условиях. Большинство наших данных относительно индуцированной фибрилляции получены в результате испытаний на животных.Очевидно, что проводить тесты индуцированной фибрилляции желудочков на людях непрактично, поэтому имеющиеся данные отрывочны. О, и если вам интересно, я понятия не имею, почему женщины, как правило, более восприимчивы к электрическому току, чем мужчины!

Предположим, я положил обе руки на клеммы источника переменного напряжения с частотой 60 Гц (60 циклов, или чередование назад и вперед, в секунду). Какое напряжение потребуется в этом чистом, сухом состоянии кожи, чтобы произвести ток в 20 миллиампер (достаточно, чтобы я не мог отпустить источник напряжения)? Мы можем использовать закон Ома (E = IR), чтобы определить это: необходимо ли в этом чистом, сухом состоянии кожи производить ток в 20 миллиампер (достаточно, чтобы я не мог отпустить источник напряжения)? Мы можем использовать закон Ома (E = IR), чтобы определить это:

E = IR

E = (20 мА) (1 МОм)

E = 20 000 вольт или 20 кВ

Имейте в виду, что это «лучший случай» (чистая, сухая кожа) с точки зрения электробезопасности, и что это значение напряжения представляет собой величину, необходимую для индукции столбняка.Чтобы вызвать болезненный шок, потребуется гораздо меньше! Также имейте в виду, что физиологические эффекты любой конкретной силы тока могут значительно отличаться от человека к человеку, и что эти расчеты являются приблизительными оценками только .

Обрызгав пальцы водой, чтобы имитировать пот, я смог измерить сопротивление рук в руках всего 17000 Ом (17 кОм). Имейте в виду, что это касается только одного пальца каждой руки, касающегося тонкой металлической проволоки. Пересчитав напряжение, необходимое для возникновения тока в 20 мА, мы получим эту цифру:

E = IR

E = (20 мА) (17 кОм)

E = 340 вольт

В этих реальных условиях потребуется всего 340 вольт потенциала от одной моей руки к другой, чтобы вызвать ток 20 миллиампер.Тем не менее, все еще возможно получить смертельный удар от меньшего напряжения, чем это. При условии значительно более низкого сопротивления тела, увеличенного за счет контакта с кольцом (полоса золота, обернутая по окружности пальца, обеспечивает отличную точку контакта для поражения электрическим током) или полный контакт с большим металлическим предметом, таким как труба или металл рукояткой инструмента сопротивление корпуса может упасть до 1000 Ом (1 кОм), что приведет к тому, что даже более низкое напряжение будет представлять потенциальную опасность:

E = IR

E = (20 мА) (1 кОм)

E = 20 вольт

Обратите внимание, что в этом состоянии 20 вольт достаточно, чтобы вызвать ток в 20 миллиампер через человека: достаточно, чтобы вызвать столбняк.Помните, было высказано предположение, что сила тока всего 17 миллиампер может вызвать фибрилляцию желудочков (сердца). При сопротивлении рукопашной в 1000 Ом для создания этого опасного состояния потребуется всего 17 вольт:

E = IR

E = (17 мА) (1 кОм)

E = 17 вольт

Семнадцать вольт — это не очень много для электрических систем. Конечно, это «наихудший» сценарий с напряжением переменного тока 60 Гц и отличной проводимостью тела, но он действительно показывает, насколько низкое напряжение может представлять серьезную угрозу при определенных условиях.

Условия, необходимые для создания сопротивления тела 1000 Ом, не должны быть такими экстремальными, как то, что было представлено (потная кожа при контакте с золотым кольцом). Сопротивление тела может уменьшаться при приложении напряжения (особенно если столбняк заставляет пострадавшего крепче держать проводник), так что при постоянном напряжении удар может усилиться после первого контакта. То, что начинается как легкий шок — ровно настолько, чтобы «заморозить» жертву, чтобы она не могла отпустить ее, может перерасти в нечто достаточно серьезное, чтобы убить ее, поскольку сопротивление их тела уменьшается, а сила тока соответственно увеличивается.

Research предоставило приблизительный набор цифр для электрического сопротивления точек контакта человека в различных условиях (информацию об источнике этих данных см. В конце главы):

  • Провод, касающийся пальцем: от 40 000 Ом до 1 000 000 Ом в сухом состоянии, от 4 000 Ом до 15 000 Ом во влажном состоянии.
  • Провод в руке: от 15 000 Ом до 50 000 Ом в сухом состоянии, от 3 000 Ом до 5 000 Ом во влажном состоянии.
  • Плоскогубцы по металлу, удерживаемые вручную: от 5000 до 10000 Ом в сухом состоянии, от 1000 до 3000 Ом во влажном состоянии.
  • Контакт ладонью: от 3000 Ом до 8000 Ом в сухом состоянии, от 1000 Ом до 2000 Ом во влажном состоянии.
  • 1,5-дюймовая металлическая труба, удерживаемая одной рукой: от 1000 Ом до 3000 Ом в сухом состоянии, от 500 Ом до 1500 Ом во влажном состоянии.
  • 1,5-дюймовая металлическая труба, захватываемая двумя руками: от 500 Ом до 1500 кОм в сухом состоянии, от 250 Ом до 750 Ом во влажном состоянии.
  • Рука погружена в токопроводящую жидкость: от 200 Ом до 500 Ом.
  • Нога погружена в проводящую жидкость: от 100 Ом до 300 Ом.

Обратите внимание на значения сопротивления для двух условий с металлической трубой диаметром 1,5 дюйма. Сопротивление, измеренное при захвате трубы двумя руками, составляет ровно половину сопротивления при захвате трубы одной рукой.

Двумя руками площадь соприкосновения с телом вдвое больше, чем с одной рукой. Это важный урок: электрическое сопротивление между любыми контактирующими объектами уменьшается с увеличением площади контакта при прочих равных условиях. Когда две руки держат трубку, электроны имеют два параллельных пути, по которым они проходят от трубки к телу (или наоборот).

Как мы увидим в более поздней главе, параллельные пути цепи всегда приводят к меньшему общему сопротивлению, чем любой отдельный путь, рассматриваемый отдельно.

В промышленности 30 вольт обычно считается консервативным пороговым значением для опасного напряжения. Осторожный человек должен рассматривать любое напряжение выше 30 вольт как опасное, не полагаясь на нормальное сопротивление тела для защиты от поражения электрическим током. При этом, по-прежнему, держать руки в чистоте и сухости и снимать все металлические украшения при работе с электричеством — это отличная идея. Даже при более низком напряжении металлические украшения могут представлять опасность, поскольку проводят ток, достаточный для ожога кожи, при контакте между двумя точками в цепи.Металлические кольца, в частности, были причиной более чем нескольких ожогов пальцев из-за замыкания между точками в низковольтной и сильноточной цепи.

Кроме того, напряжение ниже 30 может быть опасным, если его достаточно, чтобы вызвать неприятное ощущение, которое может привести к рывку и случайному контакту с более высоким напряжением или другой опасностью. Я вспоминаю, как однажды жарким летним днем ​​работал над автомобилем. На мне были шорты, моя голая нога касалась хромового бампера автомобиля, когда я затягивал контакты аккумулятора.Когда я прикоснулся металлическим ключом к положительной (незаземленной) стороне 12-вольтовой батареи, я почувствовал покалывание в том месте, где моя нога касалась бампера. Сочетание плотного контакта с металлом и моей вспотевшей кожи позволило почувствовать шок всего лишь при напряжении 12 вольт.

К счастью, ничего страшного не произошло, но если бы двигатель работал и удар ощущался в моей руке, а не в ноге, я мог бы рефлекторно толкнуть руку на пути вращающегося вентилятора или уронить металлический ключ на клеммы аккумулятора ( выдача больших величин тока через гаечный ключ с большим количеством сопутствующих искр).Это иллюстрирует еще один важный урок, касающийся электробезопасности; этот электрический ток сам по себе может быть косвенной причиной травмы, заставляя вас подпрыгивать или спазмировать части вашего тела в опасную для вас сторону.

Ток, проходящий через человеческое тело, имеет значение, насколько он опасен. Ток будет влиять на все мышцы, находящиеся на его пути, а поскольку мышцы сердца и легких (диафрагмы), вероятно, являются наиболее важными для выживания, пути удара, проходящие через грудную клетку, являются наиболее опасными.Это делает путь электрического тока из рук в руки очень вероятным способом получения травм и летального исхода.

Во избежание подобных ситуаций рекомендуется работать с цепями под напряжением, находящимися под напряжением, только одной рукой, а вторую руку держать в кармане, чтобы случайно ни к чему не прикоснуться. Конечно, всегда безопаснее работать в цепи, когда она отключена, но это не всегда практично или возможно. При работе одной рукой, как правило, предпочтение отдается правой руке по двум причинам: большинство людей правши (что обеспечивает дополнительную координацию при работе), а сердце обычно находится слева от центра в грудной полости.

Для левшей этот совет может быть не лучшим. Если такой человек недостаточно скоординирован с правой рукой, он может подвергнуть себя большей опасности, используя ту руку, с которой ему меньше всего комфортно, даже если электрический ток, протекающий через эту руку, может представлять большую опасность для его сердца. Относительная опасность между сотрясением одной рукой или другой, вероятно, меньше, чем опасность работы с менее чем оптимальной координацией, поэтому выбор руки для работы лучше всего оставить на усмотрение человека.

Лучшая защита от ударов цепи под напряжением — это сопротивление, а сопротивление может быть добавлено к телу с помощью изолированных инструментов, перчаток, обуви и другого снаряжения. Ток в цепи является функцией доступного напряжения, деленного на общее сопротивление на пути потока. Как мы рассмотрим более подробно позже в этой книге, сопротивления имеют аддитивный эффект, когда они сложены так, что у электронов есть только один путь:

Теперь мы рассмотрим эквивалентную схему для человека в изолированных перчатках и ботинках:

Поскольку электрический ток должен проходить через ботинок и , тело и перчатки, чтобы замкнуть цепь обратно к батарее, общая сумма ( сумма ) этих сопротивлений противодействует потоку электронов в большей степени, чем любая другая. сопротивлений рассматривается индивидуально.

Безопасность — одна из причин, по которой электрические провода обычно покрывают пластиковой или резиновой изоляцией: чтобы значительно увеличить сопротивление между проводником и тем или иным предметом, который может с ним контактировать. К сожалению, было бы непомерно дорого заключать проводники линии электропередачи в достаточную изоляцию, чтобы обеспечить безопасность в случае случайного контакта, поэтому безопасность поддерживается за счет того, что эти линии держат достаточно далеко вне досягаемости, чтобы никто не мог случайно прикоснуться к ним.

Обзор

  • Вред для тела зависит от силы электрического тока.Более высокое напряжение позволяет производить более высокие и опасные токи. Сопротивление противостоит току, поэтому высокое сопротивление является хорошей защитой от ударов.
  • Обычно считается, что любое напряжение выше 30 может создавать опасные ударные токи.
  • Металлические украшения определенно плохо носить при работе с электрическими цепями. Кольца, ремешки для часов, ожерелья, браслеты и другие подобные украшения обеспечивают отличный электрический контакт с вашим телом и сами могут проводить ток, достаточный для возникновения ожогов кожи даже при низком напряжении.
  • Низкое напряжение может быть опасным, даже если оно слишком низкое, чтобы вызвать травму электрическим током. Их может быть достаточно, чтобы напугать жертву, заставив ее отпрянуть и коснуться чего-то более опасного в непосредственной близости.
  • Когда необходимо работать в «живой» цепи, лучше всего выполнять работу одной рукой, чтобы предотвратить смертельный путь электрического тока из рук в руки (через грудную клетку).

Как действует электрический ток на человеческое тело? — Определение и значение

Когда через тело проходит электрический ток, нервная система поражается электрическим током.Интенсивность удара зависит в основном от силы тока и пути, пройденного током через тело, а также от продолжительности контакта. В крайних случаях шок вызывает нарушение нормальной работы сердца и легких, что приводит к потере сознания или смерти.

Считается, что ток ниже 5 мА не опасен. Ток от 10 до 20 мА опасен, потому что больной теряет мышечный контроль. Сопротивление человеческого тела между двумя руками или между ногами составляет от 500 мА до 50 кОм.Если сопротивление человеческого тела принято равным 20 кОм, то контакт с питанием 230 В может быть потенциально смертельным, 230/20 000 = 11,5 мА.

Ток утечки I = E / R, где E — напряжение питания, а R — сопротивление корпуса. Сопротивление сухого тела варьируется от 70 кОм до 100 кОм на квадратный см, но когда человеческое тело влажное, оно значительно уменьшается до 700-1000 Ом на квадратный см. (Кожное сопротивление тела высокое, но внешнее сопротивление низкое).

Чтобы подчеркнуть влияние мокрого тела, можно сказать, что питание 100 В влажного тела так же опасно, как 1000 вольт, когда тело сухое.

Эффект передачи тока от руки к руке и от ноги к ноге

Ниже приводится влияние тока, проходящего от руки к руке и от ноги к ноге.

  1. Ощущение поражения электрическим током составляет около 1 мА. На этом уровне человек ощущает легкое возбуждение при контакте с электрическим полем.
  2. Максимальный ток, при котором человек может отпустить проводник с помощью мышц, непосредственно затронутых током, называется « Let Go Current ».Этот ток принимается равным 9 мА для мужчин и 6 мА для женщин.
  3. Если текущий уровень выше, чем « Let Go Current », то человек теряет способность контролировать свои мышцы, и такие токи трудно переносить. Эти токи находятся в диапазоне 20–100 мА. Эти токи вызывают физические травмы, однако сердечная и респираторная функции сохраняются в норме.

Если ток превышает 100 мА, несчастный случай может быть смертельным, поскольку насосное действие сердца прекращается и пульс исчезает.Как только сердце перестает перекачивать кровь, мозг начинает умирать, и после того, как он лишается насыщенной кислородом крови. При очень высоком токе порядка 6 мА и выше возникает опасность паралича дыхания и ожогов.

Воздействие электрического шока, вызванного переменным и постоянным током, может быть разным. Переменный ток на разумных частотах (25-60c / с) более опасен, чем постоянный ток с тем же среднеквадратичным значением.

Увеличивающееся использование высокочастотного оборудования дополнительная опасность возникает из-за прохождения высокочастотного тока через тело.При частоте около 100 гц / с ощущение шока начинает исчезать. Серьезные внутренние ожоги могут оказаться опасными. Убивает течение.

Изменение напряжения 50 В может вызвать опасный ток 50 мА. Люди пережили гораздо более высокое напряжение из-за различных факторов. Сопротивление контакта можно значительно повысить, если высушить кожу, очистить одежду и надеть обувь.

экспериментов по изучению сопротивления человеческого тела постоянному току I

Примечание редактора — Документ, на котором основана эта статья, был первоначально представлен на симпозиуме IEEE Product Safety Engineering Society в 2018 году, где был признан лучшим докладом симпозиума.Он перепечатан здесь, с разрешения, из материалов Международного симпозиума IEEE Product Safety Engineering Society по разработке соответствия продукции требованиям 2018 года. Авторские права 2018 IEEE.


Физиологические эффекты поражения электрическим током в основном индуцируются током [1], поэтому ограничения по току часто устанавливаются в стандартах безопасности для защиты человеческого тела от опасности поражения электрическим током [2]. Однако для определенных стандартов или приложений часто предпочтительны пределы напряжения.В таких случаях импеданс человеческого тела можно использовать для оценки предельного напряжения на основе безопасных пределов тока. Кроме того, импеданс человеческого тела можно использовать для построения моделей электрических цепей, представляющих пути проводимости через человеческое тело, для оценки токов прикосновения. Например, согласно UL 101 [2], импеданс человеческого тела моделируется сопротивлением 1500 Ом, подключенным параллельно конденсатору 0,22 мкФ, включенному последовательно с другим резистором на 500 Ом. Такая измерительная схема используется для оценки силы тока прикосновения при воздействии шока на уровне восприятия для синусоидального переменного тока частотой 60 Гц.

Применение

постоянного тока, особенно с опасным напряжением, становится все более распространенным из-за более широкого использования возобновляемых источников энергии (например, фотоэлектрических), систем накопления энергии и т. Д. Поэтому полезно проанализировать сопротивление человеческого тела при постоянном токе, чтобы лучше понять его влияние на физиологические эффекты поражения электрическим током для приложений постоянного тока. В стандарте IEC 60479-1 [3] указано сопротивление человеческого тела постоянному току для сопротивления человеческого тела от 25 В до 1000 В. Однако значения сопротивления постоянному току, включенные в [3], были основаны на экспериментальных данных, проведенных только при 25 В, а остальные значения значений, математически экстраполированных на основе сопротивления человеческого тела переменному току.Это делает предположение, что импеданс тела изменяется одинаково с постоянным током, как и с переменным током, что может или не может быть так. Кроме того, сопротивления корпуса постоянному току, приведенные в [3], относятся только к сухим условиям. Насколько известно авторам, в настоящее время отсутствуют данные о сопротивлении человеческого тела постоянному току во влажных условиях, основанные непосредственно на экспериментальных наблюдениях. Для влажных условий предполагается, что значения сопротивления корпуса постоянному току, указанные в стандарте IEC [3], идентичны сопротивлению корпуса переменному току во влажных условиях при каждом напряжении.Опять же, это делает предположения, которые не подтверждаются твердо экспериментальными данными.

Поскольку постоянный ток при опасном напряжении и потенциальное воздействие таких опасностей на человека становится все более распространенным явлением, необходимо иметь исчерпывающие данные об импедансе человеческого тела при постоянном токе. Такой набор данных потребует, чтобы измерения на многих людях имели какую-либо статистическую значимость (т. Е. 50 или более). Конечной целью авторов является разработка типичных значений импеданса тела постоянного тока на основе экспериментальных данных, аналогичных тем, которые доступны в настоящее время для переменного тока.

Однако пока неясно, какой уровень влияния будут иметь различные параметры измерения, а также на сегодняшний день неясно, насколько воспроизводимым будет любое данное измерение для любого конкретного человека. Авторы пришли к выводу, что поэтому было преждевременно продвигаться вперед с крупномасштабной экспериментальной программой с участием значительного числа людей-добровольцев до того, как было установлено лучшее понимание согласованности измерений на каком-либо конкретном человеке. Например, было неясно, достижимы ли воспроизводимые значения при измерении одного и того же человека в разное время в одних и тех же условиях испытаний.Насколько известно авторам, ни в одной предыдущей работе это не оценивалось. Кроме того, не было обнаружено никаких опубликованных данных о влиянии контактного материала на тестирование импеданса человеческого тела, которое, как предполагается, оказывает влияние на измеренный импеданс тела и может лучше объяснить взаимосвязь измеренного импеданса тела и приложенного напряжения. (В [3] сообщалось, что сопротивление тела уменьшается линейно по мере увеличения приложенного напряжения.) Рассмотрев эти неизвестные эффекты на измерение импеданса тела постоянного тока, авторы сделали первый шаг к дальнейшему пониманию этих факторов; результаты которого представлены здесь.Поэтому более масштабную программу испытаний пришлось отложить до проведения в качестве второго этапа нашей работы.

Первоначальная работа, о которой сообщается здесь, исследовала влияние трех переменных теста (контактный материал, влажные или сухие условия и время дня) на трех испытуемых. Материалы для контактов из меди и алюминия использовались для лучшего понимания потенциального влияния на измеренный импеданс тела и для проверки гипотезы о том, что нелинейное поведение, наблюдаемое в поведении импеданса тела, аналогично тому, что наблюдается в контактах металл-полупроводник.[6] Влажные и сухие условия использовались для оценки влияния на измеренный импеданс тела, а также для определения относительной повторяемости измерений при каждом условии. В целях безопасности эта работа ограничила объем исследования до напряжений 60 В. и ниже.

Рисунок 1: Схема измерения силы восприятия тока прикосновения


Проверка порога ударного напряжения постоянного тока

Существует четыре различных физиологических эффекта электрического шока для постоянного или переменного тока: восприятие, неспособность расслабиться, фибрилляция желудочков и ожог.Согласно экспериментам, проведенным Далзилом в 1940-х годах [1], порог поражения электрическим током постоянного тока выше, чем переменного тока. Другими словами, человеческое тело менее уязвимо для поражения электрическим током при постоянном токе по сравнению с сигналами переменного тока 50/60 Гц аналогичной величины. Что касается предела напряжения, предел постоянного тока составляет 60 В в сухих условиях и 30 В во влажных условиях, как указано в UL 1310, [4] с целью защиты от неспособности отпустить ударные воздействия. Этот предел был выбран с целью защитить 95% населения, включая детей.Обратите внимание, что этот предел определяется на основе пути от руки к обеим ногам: для других путей тока допустимый предел напряжения может быть другим. Чтобы собрать более широкий диапазон экспериментальных данных, в этой работе предел напряжения был установлен на уровне 60 В вместо 30 В как для сухих, так и для влажных условий, хотя 60 В является пределом невозможности отпускания только в сухих условиях. . Однако следует отметить, что эти пределы были установлены с учетом интересов детей, и в этой работе испытуемыми были только взрослые (и поэтому они могут выдерживать более высокие напряжения).В целях безопасности и комфорта испытуемых каждый испытуемый мог разорвать цепь в любое время во время испытания, оторвав руку от электрода (рис. 2), если ощущение восприятия становилось слишком неудобным. Ток был ограничен до уровня ниже 20 мА, как за счет настройки ограничения тока на блоке питания, так и за счет включения быстродействующего предохранителя на 20 мА, установленного последовательно с источником тока для испытуемого.

Рисунок 2: Фотография экспериментальной установки с субъектом


Оборудование и экспериментальная установка

Модель BK Precision 9183B использовалась для подачи постоянного тока во время испытаний.Во время теста выходное напряжение блока питания контролировалось портативным компьютером. Модель Dewetron DEWE-50-USB2-8 использовалась для сбора данных, включая выходное напряжение и ток от источника постоянного тока. Соединения по току и напряжению с металлическими пластинами были физически разделены, чтобы минимизировать влияние контактного сопротивления. Сопротивление тела рассчитывалось по показаниям напряжения и тока с использованием закона Ома.

На рис. 2 показана экспериментальная установка, на которой испытуемый находится в нужном положении во время тестирования.Испытуемый стоял на металлической пластине, которую чередовали между медью и алюминием. Испытуемым было предложено встать на тарелку босиком. Каждый испытуемый кладет правую руку на металлическую пластину из того же материала, что и пластина у ступней. Перед каждым сеансом тестирования руки очищали спиртовой салфеткой для удаления поверхностных масел и грязи, а также для сушки рук. Не было предпринято никаких усилий, чтобы очистить или высушить ноги. Размер металлической пластины для контакта рук составлял 100 мм на 100 мм.Этот размер определяется как «большая площадь контакта» стандартом IEC 60479-1 [2]. Согласно этому стандарту ожидается, что большая площадь контакта приведет к наименьшему сопротивлению к телу, что считается «наихудшим случаем» по сравнению с аналогичным определением «средних» и «малых» площадей контакта. В этой работе предполагается большая площадь контакта или «наихудший случай», поскольку наихудшие условия представляют наибольший интерес для приложений, связанных с безопасностью.

Известно, что на сопротивление человеческого тела влияет уровень влажности поверхности кожи [3].В этой работе поверхность рук тестировалась в двух условиях: сухой и имитирующий влажный пот и воду. Концентрация натрия в поте находится в диапазоне от 6 до 85 мэкв. на л [5], что эквивалентно от 13,8 мг / дл до 195,5 мг / дл. Для этой работы выбирается концентрация натрия на верхнем конце этого диапазона, так как это приведет к наименьшему сопротивлению и, следовательно, к «наихудшему случаю» опасности электробезопасности. Тест проводился с соленостью 80 мг-экв на литр воды, что приблизительно соответствует 95-му процентилю максимальной концентрации натрия 85 мг-экв.Это соответствует 1,85 г NaCl на литр воды. Человеческий пот также содержит калий и другие соли, но эти концентрации намного ниже по сравнению с концентрацией натрия [5]; поэтому в этом исследовании не учитывались эффекты калия.

Как показано на Рисунке 2, на тыльную сторону руки были приложены два мешка, каждый из которых был заполнен 0,5 кг металлической дроби. Испытуемые были проинструктированы расслабить руки с намерением, чтобы давление оказывалось только весом мешков.Это была попытка контролировать переменное давление на металлические пластины, которое может влиять на сопротивление контакту с телом.

Результаты

Напряжение питания подавалось линейно от 0 В до 60 В, линейно нарастало со скоростью 1 В / с. Ток был ограничен до 20 мА, и когда ток или напряжение достигали предела, источник питания переключался на источник постоянного тока на 20 мА. Каждое испытание продолжается до тех пор, пока не будет достигнуто 60 В или пока испытуемый не уберет руку с пластины из-за дискомфорта.На рис. 3 показан пример измеренных напряжения и тока в сухих условиях для одного испытуемого. В этом случае приложенное напряжение постоянного тока достигало 60 В, затем снова снижалось до 0 В, при этом наблюдаемый ток составлял около 12 мА при приложенном напряжении 60 В. На рисунке 3 ось x — это временной шаг, который — это выборочный подсчет измерений, выполненных во время 60-секундного теста.

Рисунок 3: Пример измеренного выхода и тока в сухих условиях для одного объекта

Тест 1 Тест 2 Тест 3 Тест 4
Рука Сухой Сухой мокрый мокрый
Материал электрода Медь Алюминий Алюминий Медь

Таблица 1: Условия испытаний

Были измерены четыре комбинации условий испытаний: Таблица 1 показывает условия, которые использовались для каждого испытания.Для каждого условия теста и каждого добровольца тест повторялся 20 раз в течение нескольких недель. На рисунке 4 показаны результаты прямоугольной диаграммы для измеренного тока для каждого из четырех тестов и трех добровольцев при приложенном напряжении 25 В. Верхний край внешнего прямоугольника светло-голубого цвета представляет первый квартиль (Q1), а нижний край представляет третий квартиль (Q3). Таким образом, эта внешняя часть графика представляет собой межквартильный диапазон (IQR) или средние 50% наблюдений.Внутреннее поле представляет 95% доверительный интервал. Вертикальные линии представляют собой верхние и нижние усы, которые простираются наружу, чтобы указать самые низкие и самые высокие значения в наборе данных (за исключением выбросов). Горизонтальная линия в рамке представляет собой среднее значение; кружок с крестиком представляет среднее значение.

Рисунок 4: Ток (мА) при 25 В для трех добровольцев и четырех условий испытаний


Данные на Рисунке 4 показывают, что изменчивость сопротивления тела во влажных условиях (тесты 3 и 4) была значительно меньше, чем наблюдаемая для сухих условий (тесты 1 и 2).Более того, сопротивление влажного состояния было ниже, чем соответствующего сухого состояния, что означает, что влажное состояние является худшим случаем (то есть более опасным) с точки зрения безопасности. Поскольку консерватизм обычно предпочтительнее при анализе безопасности, и тот факт, что данные о влажных условиях демонстрируют меньшую изменчивость, результаты предполагают, что в будущем тестирование импеданса тела будет проводиться только во влажных условиях.

На рис. 5 показан график измеренного тока для обоих тестов 3 (алюминий) и 4 (медь, оба проводились с использованием влажной кожи) при 5В.Было замечено, что металлический электрод влияет на измеряемый ток. Для всех испытуемых медь показывала более высокий измеренный ток при 5 В и 10 В, причем этот эффект, как было обнаружено, уменьшался с увеличением напряжения. Следует отметить, что это вряд ли связано с более высокой электропроводностью меди по сравнению с алюминием, поскольку конфигурация с четырьмя датчиками, используемая для измерения сопротивления, включает в измерение только контакт металла с кожей и не включает объемное сопротивление металла. контакт.Кроме того, любой вклад в объемное сопротивление будет наблюдаться при всех напряжениях и не будет уменьшаться при увеличении напряжения.

Рисунок 5: Коробчатая диаграмма измеренного тока для Теста 3 и Теста 4 (влажные условия) при 5 В


Метод дисперсионного анализа (ANOVA) может облегчить определение значимости фактора для конкретного выходного параметра. В этом исследовании он использовался для определения статистической значимости металлического материала электрода при измерении сопротивления тела как функции напряжения.На рис. 6 показаны значения R , рассчитанные для влияния материала электрода на измеряемый ток: более высокое значение R предполагает большее влияние на выходной параметр. Было обнаружено, что значение R высокое при низких напряжениях, затем быстро падает при увеличении напряжения с 10 В до 20 В. Это говорит о том, что обнаружено, что металлический электрод оказывает статистически значимое влияние на измеренный импеданс при напряжениях ниже 20 В. , что согласуется с наличием барьера Шоттки на границе раздела металл-скин.[6]

Рисунок 6: Значение R (в процентах), рассчитанное с помощью дисперсионного анализа ANOVA во влажных условиях, оценивающее влияние электродного материала на каждого из трех добровольцев.


На рисунке 7 показано среднее значение сопротивления напряжения для испытания 4 (влажное состояние, медный электрод). Было замечено, что сопротивление тела обычно уменьшалось с увеличением напряжения прикосновения. Также было обнаружено, что сопротивление тела имеет нелинейную зависимость от напряжения, что соответствует стандарту IEC 60479-1.В стандарте IEC 60479-1 отмечается это нелинейное поведение, а также упоминается дальнейшее его усиление при электрическом пробое кожи [3]. Природа этого нелинейного поведения не описана в IEC 60479-1. Авторы предполагают, что эту нелинейность можно объяснить в контексте барьера Шоттки, где контакт кожи с металлической пластиной образует переход металл-полупроводник, что приводит к неомическому вольт-амперному поведению. Тогда разница в измеренном сопротивлении между алюминием и медью будет зависеть от работы выхода (которая для двух металлов составляет примерно 4.3 и 4,7 эВ соответственно) [6]. Измерения с использованием дополнительных металлических поверхностей помогли бы подтвердить или опровергнуть эту гипотезу, например, проведение измерений с использованием материалов с более низкой работой выхода (например, магний, 3,7 эВ) и более высокой работой выхода (например, никель и платина, 5,2 и 5,7 эВ, соответственно. ), оба потенциальных объекта будущей работы.

Рисунок 7: Среднее значение сопротивления в зависимости от напряжения для Теста 4 (влажные условия)


Коэффициент вариации (CV) представляет собой отношение стандартного отклонения к среднему, что полезно для сравнения степени вариации измеренного сопротивления для каждого отдельного добровольца.Поскольку установление повторяемости измерения импеданса тела для одного и того же человека было ключевой целью этой работы, CV помогает количественно оценить эту изменчивость. На рисунке 8 показана CV для всех трех добровольцев, использующих медные электроды во влажных условиях (тест 4), сравнивая относительные различия данных среди добровольцев. Было замечено, что волонтер № 3 продемонстрировал гораздо больший разброс в сопротивлении тела по сравнению с двумя другими добровольцами (это также можно наблюдать на Фигуре 4). Для волонтера No.2, измеренное сопротивление тела было менее согласованным при более низких напряжениях, но по мере увеличения напряжения более 20 В CV сопротивления тела снижается примерно до 10%, что согласуется с данными добровольца № 1.

Рисунок 8: Коэффициент вариации для трех добровольцев, использующих медь во влажных условиях


Для дальнейшего исследования большего разброса, наблюдаемого в результатах добровольца № 3, данные были разделены по времени суток (утро и полдень). Утро определяется как измерения, проводимые в течение обычного рабочего дня до 12:00 по местному времени, а после полудня — как измерения, завершенные после 12:00.На протяжении этой работы, как правило, каждый день выполняли два измерения каждого испытуемого: одно утром и одно днем. За отсчетом времени понималось время, когда результаты теста были завершены и сохранены в компьютере. На рисунке 9 показан коэффициент вариации для утра (а) и после полудня (б). Как и в случае, показанном на Рисунке 8, на Рисунке 9 также показаны данные с медными электродами и кожей во влажных условиях. Интересно отметить, что CV значительно различается для волонтера No.3 между утром и днем, тенденция, которая была последовательной для этого объекта во всех приложенных напряжениях, используемых в этом исследовании. Разница для двух других добровольцев между утром и днем ​​оказалась менее значительной, особенно для добровольца № 1. Следует отметить, что для тестов, проводимых утром, разброс данных для добровольца № 3 фактически был ниже, чем для добровольца № 1. для добровольца № 2 — при напряжении менее 25 В. И утром, и днем ​​CV уменьшается с увеличением напряжения.Точная природа статистически значимых различий в утренних и дневных данных для добровольца № 3 в настоящее время неизвестна. Поскольку эта разница в поведении наблюдалась при 20 измерениях в течение нескольких недель, маловероятно, что проблемы были вызваны ошибкой измерения и, скорее, связаны с каким-либо метаболическим или другим состоянием организма, затронутым в полдень (т. Е. Во время обеда). Другая, хотя и менее вероятная, возможность — это какое-то неосознанное изменение в поведении добровольца No.3 между утром и днем, хотя было бы сложно повлиять на такое изменение постоянно в течение 20 тестовых сессий. Независимо от причины, здесь важна не конкретная причина, а общее влияние на сопротивляемость организма. Эти наблюдения действительно предполагают, что будущие измерения, возможно, придется проводить как утром, так и после обеда, и соответствующее время измерения должно быть отмечено для каждого испытуемого для будущих исследований.

Рисунок 9: Коэффициент вариации с использованием медных электродов во влажных условиях (тест 4), (a) утром (b) днем ​​


Заключение

Данные показали влияние материала контакта на измеряемое сопротивление тела и может быть связано с образованием барьера Шоттки, аналогичного тому, что наблюдается в приборах типа металл-полупроводник.Это объяснило бы природу давно известного неомического поведения импеданса человеческого тела, хотя для подтверждения этой гипотезы необходимы дальнейшие исследования. Результаты показывают, что обязательно указать состав контактных материалов, используемых для измерения импеданса тела, вместе с результатами, и что для обоих контактов следует использовать только один тип контактного материала.

Влажные условия показали более стабильные результаты испытаний на сопротивление тела, чем в сухих условиях.Учитывая это, а также тот факт, что влажные условия демонстрируют меньшее сопротивление тела, чем соответствующее сухое состояние, будущая работа будет сосредоточена на использовании только влажных условий. Это исследование также продемонстрировало, что измеренное сопротивление может значительно различаться в разное время дня, а именно утром и днем, как было исследовано здесь. Также было замечено, что эта вариация относительно времени суток наблюдалась не у всех испытуемых и имела неизвестное происхождение. Независимо от причины, результаты показывают, что время суток является потенциальной переменной для импеданса тела, и его необходимо продолжать включать в будущие исследования, желательно для получения данных в разное время суток для одного и того же добровольца.Если отделить этот эффект времени суток от данных, можно заметить, что коэффициент вариации имеет тенденцию составлять около 10%, причем более высокие значения наблюдаются при более низких напряжениях.

Экспериментальная работа, описанная здесь, показывает, что импеданс человеческого тела, измеренный для конкретного испытуемого и проведенный в одних и тех же условиях испытаний, будет повторяться с течением времени. Ожидается, что данные будут нормально распределены со стандартными отклонениями примерно 10% от среднего значения для большинства испытуемых и условий, хотя для некоторых испытуемых возможна большая вариабельность (особенно из-за изменений из-за времени суток, но пока неизвестно. происхождение).

Эти результаты подтверждают, что данные более широкой выборки добровольцев, вероятно, будут репрезентативными для сопротивления тела постоянного тока каждого человека в пределах предсказуемого уровня неопределенности, даже если на добровольце был проведен только один сеанс измерения. Однако измерения, проведенные несколько раз на дополнительных добровольцах, будут полезны для лучшего понимания влияния переменных теста на людей. Это говорит о том, что в будущей работе часть добровольцев будет предложено вернуться для повторных измерений в течение нескольких дней, в то время как более широкую популяцию можно попросить принять участие только в одной или двух тестовых сессиях (желательно в двух, одна из которых проводится утром, а другая — в одной. второй в тот же день).Для дальнейшего изучения природы неомического контактного поведения трех исходных испытуемых попросят повторить тестирование с использованием дополнительных контактных материалов. Дополнительных испытуемых также могут попросить провести тесты с использованием нескольких контактных материалов. Предполагается, что для всех испытаний будут использоваться только влажные условия, поскольку измеренные токи были выше, а изменчивость данных была ниже. Ожидается, что испытания будут продолжены с тем же путем тока тела (правая рука к обеим ногам), хотя было бы полезно провести дополнительные исследования с другими путями тока тела.


Список литературы
    1. C.F. Далзил, Э. Огден и К. Эбботт, «Влияние частоты на отпускаемые токи», Труды Американского института инженеров-электриков,
      том. 62 1943.
    2. UL 101, «Ток утечки для устройства», Underwriters Laboratories Inc.
    3. Технический комитет МЭК 64, рабочая группа 4, «Воздействие электрического тока на людей и домашний скот —
      Часть 1: Общие аспекты», IEC 60479-1.
    4. UL 1310, «Блок питания класса II», Underwriters Laboratories LLC.
    5. И. Шварц и др., «Экскреция натрия и калия в человеческом поту», осеннее собрание Американского физиологического общества, стр. 114-119, Мэдисон, Висконсин.
    6. Р. Стейм, Ф. Рене Коглер и Кристоф Дж. Брабек, «Интерфейсные материалы для органических солнечных элементов»,
      J. Mater. Chem., 2010, 20, 2499-2512.
    7. Р. Т. Тунг, (2014). «Физика и химия высоты барьера Шоттки», Обзоры прикладной физики , 1 (1).

Хай Цзян получил докторскую степень.D. и магистр электротехники Дейтонского университета (Огайо). В настоящее время он является старшим инженером-исследователем и глобальным экспертом по поражению электрическим током и токам утечки в Underwriters Laboratories (UL). Цзян является старшим членом Общества IEEE и профессиональным инженером в США. Он также является основным назначенным инженером (инженер по стандартам UL) по току утечки UL101 для устройств. С Цзянем можно связаться по адресу [email protected].

Пол У.Бразис младший . является менеджером по исследованиям и заслуженным членом технического персонала UL Corporate Research в UL LLC (Нортбрук, Иллинойс, США). Он имеет опыт работы в области электрических и тепловых характеристик, электронных материалов и физики устройств, получил степень бакалавра, магистра и доктора наук в области электротехники в 1995, 1997 и 2000 годах соответственно в Северо-Западном университете (Эванстон, Иллинойс, США). Бразис присоединился к UL в 2008 году и возглавляет группу электрических и механических исследований. С ним можно связаться по адресу [email protected].

Body Resistance — A Review

Underwriters Laboratories провела фундаментальное исследование в области безопасности и опубликовала результаты этого исследования в серии «Бюллетеней исследований». Было опубликовано не менее 58 бюллетеней о пожарах, взрывах и поражении электрическим током. Один из этих бюллетеней «Удар электрическим током в отношении электрического забора» является классическим документом в области безопасности продукции. Исследование проводилось с 1936 по 1939 год Бароном Уитакером, помощником инженера-электрика в UL.В конечном итоге Уитакер стал президентом UL.

Исследования Уитакера все еще актуальны. Хотя аналогичные исследования проводились в поддержку современных публикаций МЭК, такие исследования обычно публикуются только в документах комитета МЭК и обычно в значительной степени сосредоточены на конкретном стандарте или отчете, который МЭК пытается написать.

Этот Бюллетень исследований UL по электрическому забору содержит много информации, которая относится не только к электрическому забору. Поэтому это классическая работа.

ВВЕДЕНИЕ

Уитакер представляет свое исследование, описывая электрический забор: «Одно из самых последних и новых применений электричества в сельской местности сегодня — это электрическое ограждение. Физически электрический забор отличается от обычного типа забора из колючей или плетеной проволоки тем, что он имеет более простую конструкцию (обычно с одной проволокой) и не требует механической прочности или устойчивости старых типов. Функционально он отличается тем, что управляет животными с помощью страха, а не силы или причинения боли.Электрический забор состоит из двух отдельных частей, а именно из проволоки забора и электрического контроллера, который подает электроэнергию к проволоке забора ».

НАЗНАЧЕНИЕ И УСЛОВИЯ

Уитакер изучил отчеты о несчастных случаях, связанных с электрическим забором, включая травмы от поражения электрическим током и смерть в результате поражения электрическим током. Основываясь на своем исследовании, он ограничил свое исследование тем, что заявил, что электрический забор должен быть безопасным для двухлетнего ребенка: «… босиком, стоя в луже с водой или грязью, падая или хватаясь за провод двумя мокрыми руками. или потные руки, проволока, насколько ребенок знает, представляет собой обычную неэлектрифицированную проволоку для забора.”

(Анекдотично, коллега, чей дом является фермой и использует электрические заборы, сообщил, что это были условия, в которых его жена спасла его дочь!)

В этих условиях Уитакер хотел определить максимальное значение тока как для переменного, так и для постоянного тока, частоту и продолжительность, которые «можно рассматривать как неопасные для жизни человека».

Уитакер обязался определить значения для:

  1. Электрическое сопротивление корпуса
  2. Безопасное напряжение холостого хода
  3. Влияние постоянного, прерывистого постоянного, переменного тока и частоты переменного тока
  4. Максимальный ток и продолжительность, которые не вызовут телесных повреждений
  5. Минимальное время выключения

ПРИРОДА ПОРАЖЕНИЯ ЭЛЕКТРИЧЕСКИМ ТОКОМ

Уитакер исследовал различные причины смерти от электроэнергии.Он нашел пять разных причин:

  1. Паралич дыхательных мышц, приводящий к смерти от асфиксии
  2. Кровоизлияние, вызванное повышением артериального давления при прохождении электрического тока
  3. Сердечная недостаточность, вызванная фибрилляцией желудочков
  4. Дыхательная недостаточность, вызванная нервным торможением или фактическим повреждением нервной системы
  5. Ожоги кожи и кожи с осложнениями

Исследование Уитакера было направлено на предотвращение одной или нескольких из этих травм.Он не занимался предотвращением ощущений или предотвращением рефлексивных действий, как мы это делаем в сегодняшних продуктах. Кроме того, производители электрических ограждений утверждали, что эффективное ограждение должно обеспечивать ток, достаточный для сокращения мышц, и что период отключения должен быть как можно короче.

ЭЛЕКТРИЧЕСКОЕ СОПРОТИВЛЕНИЕ КУЗОВА

Уитакер начинает рассмотрение сопротивления тела с утверждения: «Для установления безопасных рабочих характеристик контроллеров электрических ограждений необходимо учитывать человеческое тело как проводник электричества.”

Whitaker инициировал серию испытаний в UL для измерения сопротивления тела. Уитакер утверждает, что «внешняя оболочка… обеспечивает наибольшее сопротивление…» и что высокое напряжение контроллера ограждения снижает сопротивление кожи. Но Уитакер не мог подавать высокое напряжение неограниченного тока на своих испытуемых, чтобы «сломать» сопротивление кожи. Поэтому эксперименты Уитакера включали смачивание рук и ног его испытуемых 20% -ным раствором хлорида натрия.

Уитакер обнаружил, что при постоянной площади, постоянном давлении и влажных руках сопротивление тела не зависит от тока, когда ток находится в диапазоне от 1 до 15 миллиампер.

Испытательная установка

Whitaker состояла из источника постоянного тока 12 В (сухие элементы), потенциометра, вольтметра и амперметра. В качестве ручных электродов использовались провода № 10 AWG. Ножной электрод представлял собой квадратную медную пластину диаметром 14 дюймов. Потенциометр был отрегулирован на 5 миллиампер для взрослых и 1 миллиампер для детей. Было измерено напряжение на объекте и рассчитано сопротивление.

Уитакер измерил 40 взрослых и 47 детей (в возрасте от 3 до 15 лет). Он обнаружил, что для взрослых «нет никаких тенденций или взаимосвязей между сопротивлением тела людей и их полом, возрастом, ростом или весом.«Я представил гистограммы различных измерений Уитакера на рисунках 1 и 2.

Рисунок 1

Рисунок 2

На основании этих данных он пришел к выводу, что «наименьшее сопротивление тела, с которым можно было бы считаться в связи с применением электрического забора, будет не менее 500 Ом».

(В более позднем Бюллетене исследований эти же данные используются Карлом Гейгесом для разработки печально известного измерителя тока утечки. Я рассмотрю работу Гейгеса в одном из следующих выпусков.)

НАПРЯЖЕНИЕ

Уитакеру потребовалось определить два фактора относительно напряжения:

  1. Если выходной ток ограничен, нужно ли контролировать напряжение холостого хода?
  2. Если выходной ток не ограничен, какое максимальное напряжение холостого хода?

Уитакер определил, что максимальное безопасное напряжение (от источника напряжения, выходной ток которого не ограничивается) будет таким напряжением, которое не вызывает телесных повреждений и позволяет человеку освободиться от ограждения.

Уитакер сообщает о серии испытаний, проведенных сотрудниками UL в течение 1930 года, которые, кстати, регистрировали напряжение, которое человек мог выдерживать и при этом сохранять произвольный контроль над своими мышцами. Исходя из этих данных, минимальное напряжение составляло 20 вольт среднеквадратического значения.

Whitaker также сообщает об испытаниях, проведенных International Harvester Co., в которых напряжение было подключено к ведру, наполненному водой, и к ручному электроду, который держал субъект. Затем испытуемого попросили достать объект, погруженный в ведро.International Harvester обнаружила, что максимальное напряжение для извлечения объекта составляло от 12 до 20 вольт.

Уитакер пришел к выводу, что «нет необходимости ограничивать напряжение холостого хода, если устройство имеет встроенные функции ограничения тока».

Тем не менее, «если в устройстве нет встроенных функций ограничения тока, максимальное безопасное напряжение… не должно превышать 12. Это основано на теории, согласно которой потенциал 12 В или менее редко, если вообще когда-либо, вызывает пробой. сопротивления кожи, достаточного для того, чтобы позволить току через тело такой силы, чтобы вызвать потерю мышечного контроля или физическое повреждение человека.”

ЧАСТОТА

Уитакер сообщает, что «главное отличие физического воздействия постоянного тока от переменного тока состоит в том, что постоянный ток не вызывает сокращения мускулов в той степени, в какой это связано с переменным током».

Уитакер также отмечает, что Кувенховен и д’Арсонваль обнаружили, что по мере увеличения частоты ток также должен увеличиваться, чтобы иметь такой же физиологический эффект.

Тем не менее, Уитакер заключает, что «в настоящее время нет никаких оснований для разрешения более высоких значений тока… независимо от используемой частоты.”

ТОК

Из тех же данных, где Уитакер определил максимальное напряжение, и из других данных, Уитакер определил, что минимальные и максимальные значения, при которых люди сохраняют произвольный контроль над мышцами, составляют около 6 миллиампер и 20 миллиампер, соответственно.

Уитакер также изучил результаты тестов на фибрилляцию на собаках и овцах, поскольку считалось, что сердца этих животных реагируют на раздражители так же, как и люди.Из испытаний на овцах и поскольку овцы имеют вес тела и сердца, аналогичный человеческому, Уитакер определил, что минимальный ток фибрилляции прямо пропорционален массе тела и массе сердца.

Дальнейшее изучение данных тестов на овцах показало, что фибрилляция была функцией фазы сердечного цикла в момент возникновения шока и функцией продолжительности шока. Уитакер обнаружил, что фибрилляция для разряда длительностью 0,1 секунды требует в 10 раз больше тока, чем для разряда 3 секунды.

Затем

Уитакер построил график 3-секундных токов фибрилляции для различных взрослых животных в зависимости от массы тела и массы сердца. Затем Уитакер предположил, что минимальное значение такой кривой соответствует человеку. Уитакер также предположил, что минимальный ток фибрилляции для разного веса тела и сердца является постоянным соотношением, при условии, что продолжительность разряда составляет один и тот же процент сердечного цикла, и разряды инициируются в одной и той же точке сердечного цикла.

Используя эти предположения и данные, Уитакер определил, что минимальный 3-секундный ток фибрилляции для тела весом 125 фунтов составляет 126 миллиампер, а для веса тела 20 фунтов — 31 миллиампер.(За средний вес двухлетнего ребенка принимается 20 фунтов.)

Используя эти числа, Уитакер определил соотношение 31: 126 для минимальных токов фибрилляции для тела массой 20 и 125 фунтов.

Используя это соотношение и учитывая процент времени полного сердцебиения, массу тела и массу сердца, Уитакер смог построить кривую зависимости времени контакта от минимального тока фибрилляции для двухлетнего ребенка. . » Эта кривая аппроксимировала прямоугольную гиперболу.См. «График 3» Уитакера на рис. 3.

Рисунок 3

Затем Уитакер произвольно установил максимальный ток на 65 миллиампер, максимальный выход на 4 миллиампер-секунды и максимальный период включения на 0,2 секунды. Кривая зависимости времени контакта от допустимого тока была в 6 раз меньше кривой минимального тока фибрилляции. См. «График 4» Уитакера на рис. 4.

Рисунок 4

Уитакер пришел к выводу, что:

  1. максимальный безопасный непрерывный ток составляет 5 миллиампер, а
  2. максимальная продолжительность любого тока не должна превышать 4 миллиампер-секундной кривой.

ПЕРИОД ВЫКЛЮЧЕНИЯ

Во время исследования Уитакера диспетчеры ограждений производили последовательные электрические разряды с интервалом примерно в 1 секунду. Уитакеру нужно было определить минимальный период «отдыха», который позволил бы человеку освободиться от заграждения.

UL провела испытания, в ходе которых на человека внезапно оказывалось напряжение и регистрировалось время его высвобождения. Этот тест считался «непроизвольной» реакцией. Уитакер отметил, что время восприятия ощущения обратно пропорционально интенсивности стимула.

Уитакер также изучил другие тесты времени реакции. Большая часть этих других тестовых данных касалась «произвольной» реакции на такие стимулы, как прикосновение, зрительные или слуховые. Уитакер также отметил, что мышечное сокращение, связанное с постоянным током, имеет тенденцию отбрасывать жертву от проводника, в то время как мышечное сокращение, связанное с переменным током, как правило, невозможно отпустить.

Таким образом, Уитакер пришел к выводу, что период «выключения» для контроллеров переменного тока должен составлять 0,90 секунды, а для контроллеров постоянного тока должен быть 0.75 секунд.

ПРАВО

Whitaker также исследовал, может ли испуг, вызванный случайным контактом с «безопасным» забором, отрицательно повлиять на сердце или вызвать фибрилляцию. Медицинские власти, с которыми он консультировался, не смогли предсказать такое событие. Один авторитет даже зашел так далеко, что заявил, что такой слабый шок не способен вызвать ни испуга, ни удивления.

Я предлагаю извлечь некоторые уроки из работы Уитакера. Во-первых, Уитакер сосредоточился на различных травмах, вызванных поражением электрическим током, а не на соблюдении стандартов.Конечно, тогда еще не было стандартов. Сегодня, когда мы анализируем новую ситуацию с безопасностью, мы, кажется, делаем это со ссылкой на стандарт, а не на травму.

Во-вторых, Уитакер провел множество измерений, но использовал только минимальные найденные значения наихудшего случая. Такой пессимизм действительно необходим в сфере безопасности. Я слишком часто думаю, что мы склонны использовать вероятностные и нормальные распределения, а не значения наихудшего случая.

В-третьих, Уитакер делает множество предположений и произвольных решений, особенно в отношении животных, представляющих людей.Я предлагаю иметь в виду, что ценности, представленные Уитакером, неточны. Многие другие значения, которые мы используем в области безопасности, также неточны, но мы относимся к ним так, как если бы они были точными.

Наконец, я обнаружил, что мы больше не проводим таких исследований. Коллега, Дж. Ф. Кальбах, придумал термин BOGSAT, означающий «группа парней, сидящих и разговаривающих», чтобы описать, как когда-то развивалась конкретная кривая. Кривая не имела инженерной или физической основы. Чисто произвольно.Я бы предположил, что наши стандарты безопасности содержат слишком много требований от процесса BOGSAT.

БЛАГОДАРНОСТИ

Джим Пирс, испытательные лаборатории ETL, уронил копию этого бюллетеня UL на мой стол и спросил, читал ли я его. Я видел и читал Бюллетень много лет назад, поэтому его копия лежала у меня на столе много месяцев. В конце концов, я взял его в руки и начал читать. Работа произвела на меня впечатление, и я подумал, что пересмотрю ее для вас.

Я также хочу поблагодарить Генри Джонса, консультанта по безопасности продукции, за его комментарии об электрических ограждениях.Также благодарим Тима Крамера из Hewlett-Packard Company за подготовку гистограмм сопротивления тела.

Ричард Нут (Richard Nute) — консультант по безопасности продукции, занимающийся безопасным проектированием, безопасным производством, сертификацией безопасности, стандартами безопасности и судебно-медицинскими расследованиями.

Фото Томаса Фано

ток — Истинное значение сопротивления кожи

ток — Истинное значение сопротивления кожи — Электротехника
Сеть обмена стеком

Сеть Stack Exchange состоит из 178 сообществ вопросов и ответов, включая Stack Overflow, крупнейшее и пользующееся наибольшим доверием онлайн-сообщество, где разработчики могут учиться, делиться своими знаниями и строить свою карьеру.

Посетить Stack Exchange
  1. 0
  2. +0
  3. Авторизоваться Подписаться

Electrical Engineering Stack Exchange — это сайт вопросов и ответов для профессионалов в области электроники и электротехники, студентов и энтузиастов.Регистрация займет всего минуту.

Зарегистрируйтесь, чтобы присоединиться к этому сообществу

Кто угодно может задать вопрос

Кто угодно может ответить

Лучшие ответы голосуются и поднимаются наверх

Спросил

Просмотрено 4к раз

\ $ \ begingroup \ $

Когда я гуглил о сопротивлении кожи, я обнаружил, что люди упоминают значения от 500 Ом, но обычно около 10-100 кОм.Когда я измерил сопротивление собственной кожи, поместив датчики на внутреннюю сторону предплечья (чуть ниже запястья) на расстоянии 1-2 см между датчиками, сопротивление оказалось настолько высоким, что мультиметр даже не смог его измерить. Я попытался намочить руку слюной и получил сопротивление выше 1000 кОм, и да, я имею в виду 1 миллион Ом. Чтобы убедиться, что я не неправильно истолковал числа на мультиметре, я измерил резистор 100 кОм при той же настройке, и он показал мне «100», а затем снова мою кожу, и он показал мне примерно «1487».Это возможно?

Создан 18 сен.

Elec0Elec0

4722 серебряных знака66 бронзовых знаков

\ $ \ endgroup \ $ 2 \ $ \ begingroup \ $

Да, это возможно.Теперь попробуйте измерить сопротивление кожи на большей площади контакта. Также имейте в виду, что при поражении электрическим током от сети (высокого напряжения) кожа — это не просто пассивный резистор. Он может сломаться, и когда это произойдет, вы увидите сопротивление кровотока и жировой / мышечной ткани, которое значительно меньше.

Добавить комментарий

Ваш адрес email не будет опубликован. Обязательные поля помечены *